You are on page 1of 146

Financial Planning Academy

Investment Planning

Part 1

1) Which of the following costs best describes the cost of foregone income that results
from making an economic decision to use funds to purchase a piece of equipment?
(a) Cost of Capital
(b) Fixed Cost
(c) Marginal Cost
(d) Opportunity Cost

2) A mutual fund that invests in Indian Equities, foreign equities, Indian Corporate
Bonds, Indian Government Gilts is subject to the following risks?
1.Business Risk,
2.Default Risk,
3. Systematic Risk,
4. Interest Rate Risk.
(a) 1 & 3 only
(b) 1,3 & 4 only
(c) 3 & 4 only
(d) 1,2,3 & 4

3) Which of the following statements concerning technical stock market indicators is/are
correct?
1. The stock market is considered strong when the volume of the market is increasing in
a rising market.
2. 2. The market’s direction will change when the percent of odd-lot short sales
significantly increases or decreases.
3. Prices crossing the moving average line would be an indication of the change in the
market.

(a) 1 only
(b) 1 & 2 only
(c) 2 & 3 only
(d) 1,2 & 3

4) If a new issue was offered to the public at 15 times earnings but the market was pricing
similar shares at 19 times, this would be _____________.
(a) Appalling proposition to the investor
(b) The investor cannot take a position
(c) An example of low gearing
(d) Bargain not to be missed

Financial Planning Academy 1


5) Mr. X buys 50 RIL October Rs. 350 call options for Rs. 15. The current share price is
Rs. 345. The break-even share price, ignoring transaction costs is Rs.________.
(a) 350
(b) 360
(c) 365
(d) None of the above

6) Investor complaints relating to the following Capital Market issues will not be
entertained by SEBI:
(a) A company declaring no dividend on equity for the fourth consecutive year.
(b) A company has declared dividend but not paid the same after six months of
declaration.
(c) A company not paying the redemption proceeds on debentures issued by the
company, one year after maturity date.
(d) None of the above cases.

7) If a bond is selling at a premium:


(a) It is an attractive investment
(b) Its coupon rate is below market rate
(c) Its current yields is lower than the coupon rate
(d) Its realized compound yield will be less than the yields to maturity

8) A portfolio manager can hedge a share portfolio by_______.


(a) Buying call options
(b) Selling call options
(c) Buying index options
(d) Selling put options

9) Which of the following is NOT a characteristic of a balance fund?


(a) It is less risky than growth funds
(b) It is more risky than income funds
(c) It must invest in both equity and bonds in equal amount
(d) It provides both growth and income objectives

10) Beta is a measure of:


(a) Geometric average return
(b) Holding period return
(c) Systematic risk
(d) Unsystematic risk

Financial Planning Academy 2


11) If the current share price is S and the set exercise price is X, the intrinsic value of the
call option is_______.
(a) Max (O, S-X)
(b) Max (O, X-S)
(c) Min (O, S-X)
(d) Min (O, X-S)

12) The CAPM is a model that:


(a) Determines the geometric return of a security.
(b) Determines time-weighted return
(c) Explain return in terms of risk.
(d) Explains systematic risk

13) SAIL is an AAA rated issuer of Corporate Bonds in the International Debt markets.
The issue price of a typical SAIL corporate bond is affected by all the following EXCEPT
the _______.
(a) Face value, coupon rate, and maturity of the bond.
(b) Firm’s required return on debt.
(c) Percentage of debt in the firm’s capital structure.
(d) Required return on the firm’s competitors’ bonds.

14) RPL is raising funds through a bond issuance to fund a new power plant at Noida, UP.
They are issuing Two Year maturity, Zero-coupon bond with face value of Rs 1000 and
yield of 4%. What price would you pay for this RPL Zero-coupon bond today?
(a) Rs. 920.00.
(b) Rs. 924.56.
(c) Rs. 925.95.
(d) Rs. 960.00.

15) The best method of valuing a share is:


(a) Book value based on net tangible assets.
(b) Liquidation value based on the proceeds of liquidation of the company.
(c) Present value of all the dividends to be received from holding that share.
(d) Apply the P / E ratio to expected earnings per share.

16) Public Issue through the book building process is better than I.P.O at fixed price
because__________.
(a) High fixed price will result in under subscription leading to loss to the investor.
(b) It helps the issuer to ascertain the exact price at which the investor is willing to
subscribe.
(c) Low fixed price will result in over subscription leading to loss to the issuer.
(d) All of the above

Financial Planning Academy 3


17) Equal amount of investment is made in portfolio consisting of securities X and Y.
Standard deviation of X is 12.43%. ; Standard deviation of Y is 16.54%. ; Correlation
coefficient is 0.82. ; The interactive risk of the portfolio, measured by covariance is
(a) 145.64
(b) 156.22
(c) 168.59
(d) 172.56

18) The risk free return of Security A is 8%. In addition to it, you expect that the return
on market would be 14%. The expected return of Security A with beta of 0.70 is
________.
(a) 12.2%.
(b) 15.4%.
(c) 17.8%.
(d) 18.2%.

19) Portfolio A had a return of 12% in the previous year, while the market had an average
return of 10%. The standard deviation of the portfolio was calculated to be 20%, while
the standard deviation of the market was 15% over the same time period. If the
correlation between the portfolio and the market is 0.8, what is the Beta of the portfolio
A?
(a) 0.94
(b) 1.07
(c) 1.31
(d) 1.91

Beta=[(S.D of security / S.D of Market) *Coefficient of correlation]

20) Sonu Nigam has two Mortgage Loan options before him. The interest rate, and other
conditions are the same for both, except that one has a repayment term of 15 years and
the other has a repayment term of 30 years. Anil wants to evaluate the EMIs for both
terms. All other conditions being the same, repaying a loan in 15 years instead of 30
would require EMIs that are________.
(a) Half the size of the 30-year loan payments.
(b) Less than twice as large as the 30 year loan payments.
(c) More than twice as large as the 30 year loan payments.
(d) Twice as large as the 30 year loan payments.

Ex: Suppose a loan of Rs 1,00,000


Interest = 8%
EMI for 15 Years=Rs 11682.95
EMI for 30 Years=Rs 8882.74
Rs 11682.95 < 2 * 8882.74

Financial Planning Academy 4


21) You are running a Dividend Yield Fund for a leading Mutual Fund House. The most
recent dividend of All Is Fine Business Services common stock was Rs 2.35. The
dividends are expected to grow at 4 percent indefinitely. If you are looking at a 12
percent return, how much will you be willing to pay for one share of All Is Fine Business
Services?
(a) Rs. 24.79.
(b) Rs. 29.38.
(c) Rs. 29.38.
(d) Rs. 30.55.

M.V of a Stock=[(Dividend*(1+growth rate)] / (Return- growth rate)

22) Ram is an owner of an apartment complex with 300 units, each of which can fetch
Rs. 1000 p.m. as rentals. The apartment complex has an average occupancy rate of 75%.
The expenses for maintaining, up keeping the apartment comes to around Rs. 10 Lakh
p.a. Based on the concept of capitalized earning approach and assuming that you require
a capitalization rate of 10%, how much is the complex worth now?
(a) Rs. 1.50 crore
(b) Rs. 1.70 crore
(c) Rs. 2.00 crore
(d) None of the above.

Annual rent receivable =300*(1000*12)*0.75 (A)


Annual upkeep & maintenance =Rs 10 lakhs (B)
Net income annually=Rs 17,00,000 (A-B)
Capitalization rate=10%
Value of the complex=Net income / Capitalization rate

Financial Planning Academy 5


Part 2

Q1. Which of the following is/are not a method(s) to evaluate the attractiveness of
investments?
I. Net present value.
II. Internal rate of return.
III. Sum of digits.
IV. Payback period.
A. I
B. I, II
C. III
D. III, IV

Q2. Calculate the price earnings ratio (PER) of the stock of Company A, with the
following information:
Price = 4.00
Profit before tax = 66.0 million
Profit after tax = 48.0 million
Paid up Capital = 120 million
At par value of 0.50 per share
A. 7.3
B. 10.0
C. 20.0
D. 14.5

PER= Market Price / EPS


Profit after tax = 48 m
No. of shares = 120 m / 0.50 = 240 m
EPS = 48m / 240 m = 0.20
PER = 4 / 0.20 = 20x

Q3. Company A has announced a 20% increase in net profit over the previous year. The
stock price however fails to respond to the news and remains unchanged. This
phenomenon reflects:
A. Weak form of market efficiency.
B. Semi-strong form of market efficiency.
C. Strong form of market efficiency.
D. Investors are selling on fact.

Financial Planning Academy 6


Q4. Which of the following factors will lead to a higher discount rate and therefore a fall
in stock prices?
I. Increase in interest rates.
II. Increase in political risk.
III. Lower dividend rate.
IV. Lower EPS growth rate.
A. I, II
B. I, III
C. I, IV
D. III, IV

Q5. A corporation proposes to issue a 5 year bond with a coupon rate of 8.0%. The
prevailing yield to maturity of bonds with similar risk and term is 10.0%. The bond will
sell at ____________ to its face value.
A. A premium.
B. A discount.
C. At par.
D. A price predetermined.

N = 5 Pmt = 80 I = 10% FV = 1,000 Compute PV = 924.18, a discount to face value

Q6. Intervention by RBI in the money market through buying of government securities
has resulted in MIBOR rates falling across the board. If a unit trust bond fund marks to
market all its bonds, the price of the unit trust fund should _____________.
A. Remain unchanged.
B. Declare a dividend.
C. Appreciate.
D. Depreciate.

Q7. Which among the following is/are advantages of investing in unit trust funds?
I. Management is professional.
II. Diversification from a larger portfolio of stocks.
III. Affordability.
IV. Ease of conversion – liquidity.
A. I
B. I, II
C. I, II, III
D. All of the above.

Financial Planning Academy 7


Q8. An investor invested 1,200 in Fund A – an equity growth fund. The prices quoted by
the XYZ AMC were as follows:
Buy Sell
0.95 1.00
When the investor received his statement, he found that he had bought 1,188 units. Which
of the following is a plausible reason?
A. The stock market appreciated.
B. The stock market depreciated.
C. The unit trust company uses forward pricing.
D. The unit trust company uses same day pricing.

HINT:-
1,200 / 1.00 = 1,200 units against 1,1888 units
The unit trust company uses forward pricing.

Q9. Which of the following are major reasons for making real estate investments?
I. Reasonably liquid.
II. Comparatively higher leverage.
III. Good hedge against inflation.
IV. Higher capital gains.
A. I, II
B. II, III
C. III, IV
D. I, IV

Q10. Given the following information, what would be a reasonable rental for House D?
Comparable Houses Net Monthly Rental Market Value () Market
in Subang Jaya Income () Capitalization Rate
(%)

House A 1500 327272 5.50 %

House B 1400 336000 5.00 %


House C 1250 285714 5.25 %
House D ? 280000

A. 1,175.00
B. 1,225.00
C. 15,300.00
D. 14,700.00

Financial Planning Academy 8


HINT :-
House A = (1,500 x 12) / 327,272 = 5.50%
House B = (1,400 x 12) / 336,000 = 5.00%
House C = (1,250 x 12) / 285,714 = 5.25%
House D = get average = (5.50% + 5.00% + 5.25%) / 3 = (15.75% / 3) = 5.25% (Net
Monthly Rental x 12) / 280,000 = 5.25%
Net Monthly Rental x 12 = 5.25% x 280,000 = 14,700
Net Monthly Rental therefore is 14,700 / 12 = 1,225

Q11. Diversification is most effective when the returns on securities are:


A. Positively correlated.
B. Zero correlated.
C. Negatively correlated.
D. High correlation.

Q12. The returns on Stock A and Stock B have a correlation coefficient of –1. When the
price of Stock A appreciates by 12%, how will Stock B’s price perform?
A. Appreciate by 12%.
B. Depreciate by 12%.
C. Stay unchanged.
D. Depreciate by 6.0%.

Q13. A preference share has preference over a common stock for the reason that in a
liquidation scenario, it ranks above____________.
A. Bank loans.
B. Bonds.
C. Convertible bonds.
D. Common stocks.

Q14. A call warrant with a strike price of 2.30 is currently trading at 2.00. The underlying
stock price is quoted at 4.40. What is the time value of the warrant?
A. 0.00
B. 0.30
C. 2.10
D. 2.40

HINT: -
Call Value = Time Value + Intrinsic Value
Intrinsic Value (IV) = Stock Price – Strike Price
IV = 4.40 – 2.30 = 2.10 (deep in-the-money warrant)
The deep in the money warrant has zero Time Value

Financial Planning Academy 9


Q15. Company A has just declared a 1:2 Bonus Issue followed by a 2:5 Rights Issue at
1.10 per share. (The bonus shares are entitled to the Rights.) The current market price of
Stock A is 5.80 per share. Calculate the ex-all price.
A. 2.98
B. 3.00
C. 3.04
D. 3.08
HINT: -
Existing 5.80 x 1 = 5.80
BI 0 x 0.5 = 0
RI 1.10 x 0.6 = 0.66
2.1 6.46

6.46/2.1 = 3.08
Note: since the bonus shares are entitled to the Rights, 0.6 is derived from =
(2/5 x 1 existing) + (2/5 x 0.5 from bonus)

Financial Planning Academy 10


Part 3

At the beginning of the year Anurag decided to take Rs 50000 In savings out of bank
deposits and invest it in a portfolio of stocks and bonds; Rs 20000 was placed into
common stocks and Rs 30000 into corporate bonds. A year later, Anurag’s stock and bond
holdings were worth Rs 25000 and Rs 23000 respectively. During the year Rs 1000 cash
dividend was received on the stocks and Rs 3000 in coupon payments was received on
the bonds. The stock and bond income was not reinvested in Anurag’s portfolio.

1 Find out the return on Anurag’s stock portfolio?


(a) 30.5%
(b) 28.9%
(c) 30%
(d) 25%

2 What was the return on Anurag’s bond portfolio?


(a) 10%
(b) 8.95%
(c) –13.3%
(d) –12.5%

3 Find out the return on Anurag’s total portfolio during the year?
(a) 3%
(b) 4%
(c) 4.5%
(d) 5%

Determine the expected return on the following

SECURITIES NO OF SHARES COST PRICE EXPECTED


YEAR END PRICE
A 200 100 140
B 150 75 78
C 300 125 140
D 100 65 95

4 What is the return on Security B


(a) 4%
(b) 5%
(c) 2%
(d) 3.5%

Financial Planning Academy 11


5 The expected return on Security C is
(a) 10.5%
(b) 12%
(c) 6%
(d) 8%

6 Determine the return on security A


(a) 40%
(b) 20%
(c) 28%
(d) 30%

7 What is the return on the portfolio?


(a) 20%
(b) 21%
(c) 18%
(d) 19%

Shown here are the following data on two companies in the same industry.
COMPANY Market Price per Dividend per Share Earnings per share
share
A Ltd 60 10.00 17.50
B Ltd 40 12.00 22.00

8 What is the dividend yield of A Ltd


(a) 16.67%
(b) 15.00%
(c) 16.50%
(d) 17%

9 What is the dividend yield of B Ltd


(a) 32%
(b) 25.50%
(c) 28.00%
(d) 30%

10 P/E Ratio of A Ltd is


(a) 3.43
(b) 3.80
(c) 3.40
(d) 3.00

Financial Planning Academy 12


11 P/E Ratio of B Ltd is
(a) 1.82
(b) 2.00
(c) 1.50
(d) 1.60

12 Rahul Mehra proposes to purchase a property for giving it on rent. He expects to


receive Rs 55000 in net receipts each year for six years and to sell the property for Rs
8,50,000 at the end of the six-year period. If the expected return is 15% what would
be the value of the property?
(a) Rs 800000
(b) Rs 756700
(c) RS 575625
(d) Rs 448678

13 For an asking price of a property at Rs 9,50,000 with an estimated net income of Rs


1,25,000 at a market yield of 12%, calculate the value of the property on
capitalization approach.
(a) 10,42,000
(b) 12,56,000
(c) 9,88,888
(d) 8,98,850

14 With the following data shown in the table below, compute the risk on the portfolio.
SECURITY Std Deviation Proportion
A 14.5% 60%
B 18.5% 40%
Corr Coeff 0.91

(a) 14.50
(b) 15.74
(c) 16.31
(d) 14.78

Financial Planning Academy 13


Calculate for security X and Y based on the data given below
Probability Security X Security Y
0.1 40 40
0.2 20 30
0.4 0 15
0.2 -5 0
0.1 -10 -20

15 What us the rate of return on Security X


(a) 6%
(b) 5.5%
(c) 6.5%
(d) 5.

16 What is the return on security Y


(a) 12%
(b) 14%
(c) 15%
(d) 13%

17 Calculate Standard deviation(risk) on the portfolio assuming 50% weight of each


security.
(a) 18.83
(b) 17.50
(c) 15
(d) 15.20
(e) 17.00

18 Calculate Standard deviation of X


(a) 14.8%
(b) 14.5%
(c) 13.8%
(d) 15.25%

19 Calculate Standard deviation of Security Y


(a) 16.00%
(b) 16.55%
(c) 18.50%
(d) 17.25%

20 Calculate the Covariance (security is not mentioned)


(a) 220
(b) 212
(c) 216
(d) 220

Financial Planning Academy 14


21 For the Financial Year 2006-2007 what is true about the dividend on equity schemes?
(a) The recipient is required to pay tax at normal rates.
(b) The recipient is not required to pay any tax on dividends.
(c) The distributing company is required to pay distribution tax.
(d) None of these.

22 For the Financial Year 2006-07, what is true about dividends on debt schemes?
(a) The distributing company is required to pay dividend distribution tax.
(b) The recipient has to pay tax on dividend received by him.
(c) The recipient is required to pay tax at marginal rates.
(d) None of these.

23 Which of the following statements is true.


(a) Price of an option is called as premium.
(b) A short call is buying a call possibly with the hope of selling it back later at lower
price.
(c) A short call is selling a call possibly with the hope of buying it back at higher price.
(d) A short call is selling a call possibly with the hope of buying it back later at lower
price.
(e) Both (a) and (d)

24 Which of the following statement is not true?


(a) Buying a call option allows the taker to profit from an increase in the price of
underlying securities
(b) Buying a call option allows the taker to profit from a decrease in the price of
underlying securities.
(c) Call options whose strike price exceeds the spot price are called IN THE MONEY
options.
(d) Call options whose spot price exceeds the exercise price are called IN THE
MONEY options.
(e) Both (b) and (d)

25 Which of the following statements are true about future contracts?


(a) Forward prices are normally higher than spot prices.
(b) Usually the positions are marked to market.
(c) Daily settlement takes place.
(d) Both (a) and (b)
(e) (a),(b) and (c)

26 Intrinsic value is
(a) Difference between original and current share price.
(b) Difference between original and strike price.
(c) Difference between strike and current share price.
(d) None of these

Financial Planning Academy 15


27 A stock currently sells at 120. The put option to sell the stock sells at Rs134 costs
Rs 18 .The time value of the option is---------
(a) Rs 18
(b) Rs 4
(c) Rs 14
(d) Rs 12.

28 An in-the-money option would generate


(a) Positive cash flow
(b) Pre determined amount of cash flow
(c) No cash flow
(d) Negative cash flow

29 A put option gives the ------------ the right to but not the obligation to --------------the
underlying asset at a specified price.
(a) Seller, buy
(b) Seller, sell
(c) Owner, buy
(d) Owner, sell

30 By buying index futures one can make------------------------


(a) Unlimited profits or loss since market may go up or down.
(b) Limited profit but unlimited losses.
(c) Limited profits or losses.
(d) Unlimited profit but limited loss.

31 A call option at a strike price of Rs 176 is selling at a premium of Rs 18. At what


price will it break even for the buyer of the option?
(a) Rs 196
(b) Rs 204
(c) Rs 187
(d) Rs 194

32 Typically option premiums is


(a) Less than the sum of intrinsic value and time value.
(b) Greater than the sum of intrinsic value and time value.
(c) Equal to the sum of intrinsic value and time value
(d) Independent of intrinsic value and time value

33 Spot value of S&P CNX Nifty is 1200. An investor bought a one –month S&P Nifty
1220 call option for a premium of Rs 10. The option is
(a) In-the money
(b) At –the- money
(c) Out-of-money
(d) None of these

Financial Planning Academy 16


34 For knowledge companies, the core activity must provide-------of the total income in
the preceding------- years
(a) Not less than 51%;3
(b) Not less than 75%;3
(c) Not less than 75%;2
(d) Not less than 51%;2

35 For listing, the company must submit to------audited balance sheets for the past----
years.
(a) SEBI;3
(b) CLB;2
(c) NSE;3
(d) ROC;2

36 For IPO’s by companies, the paid up equity must be more than------Cr and market cap
more than ------Cr
(a) 25;50
(b) 10;25
(c) 10;50
(d) 25;50

The Managing Director of ABC Ltd wants to float an IPO. He has come to know about
Book Building and is favorably inclined. However he is also concerned about the retail
investors. He does an analysis of the merits and demerits of Plain Vanilla IPO and Book
Building methods.

37 Which of the following statement is true?


(a) In book building demand for the IPO is known at the end of the issue.
(b) In book building demand for the IPO is known at the beginning of the issue.
(c) In book building demand for the IPO is known throughout the issue
(d) In book building demand for the IPO Is not known.

38 Which statement is true?


(a) In a Vanilla IPO , price discovery is better than in a book building issue.
(b) In a Vanilla IPO, price discovery is less efficient than in a book building issue.
(c) In a Vanilla IPO ,price discovery is throughout the issue.
(d) In Book building, price discovery is non-existent.

39 Which statement is true?


(a) Book building is more common in developed than emerging markets.
(b) Book building is less common in developed than emerging markets.
(c) Book building is very common in India.
(d) None of these

Financial Planning Academy 17


40 A bond has a face value of Rs 1000 and coupon rate of 8%. Five years are remaining
to maturity and required rate of return is 6%. What is the value of the bond?
(a) Rs 1084
(b) Rs 1000
(c) Rs 925
(d) Rs 950

41 In the above question if the required rate of return is 10%,the value of bond will be
(a) Increase by Rs 160
(b) Decrease by Rs 160
(c) There will be no change
(d) Value will be Rs 950

42 In case of fixed income securities, which of the following statements is not true?
(a) Current yield includes only the coupon if the security is sold immediately.
(b) Current yield includes both coupon & capital gain/loss if the security is sold
immediately.
(c) YTM is the return an investor would receive if the security were held to maturity.
(d) All are true
(e) All are false

43 NAV of one unit of a mutual fund is Rs 11. The entry load is 4%. The cost to the
investor would be
(a) Rs 11
(b) Rs 11.44
(c) Rs 10.56
(d) Rs 11.50

44 Which of the following features are present in Exchange Traded Funds


(a) Real Time NAV
(b) Daily/Real Time Portfolio Disclosure
(c) Low cost intra day trading possible
(d) All of these
(e) None of theses

45 Real Estate market in India is


(a) Highly organized
(b) Disorganized
(c) Is Free from Government controls
(d) Offers homogeneous product

46 What ails the Indian Housing Society?


(a) Lack of clear titles in most cases
(b) High stamp duty rates
(c) Obsolete tenancy & rental control laws
(d) All of the above

Financial Planning Academy 18


Assume that you own two securities with the following expected returns and standard
deviations. The proportion of holding is also indicated.
Security Expected return % Standard deviation Proportion %
A 12 15 40
B 15 20 60

47 What is the risk of the portfolio when the correlation between securities is +1.0?
(a) 12%
(b) 18%
(c) 15%
(d) 6%

48 What is the risk of the portfolio when the correlation between securities is –1.0?
(a) 12%
(b) 18%
(c) 15%
(d) 6%

49 Which policy works better in Flat but Fluctuating markets?


(a) Buy & Hold policy
(b) Constant Mix Policy
(c) CPPI
(d) None of these

50 Which policy works better when the market is moving in only one direction either up
or down?
(a) Drifting Asset Allocation
(b) Balanced Asset Allocation
(c) Dynamic Asset Allocation
(d) All of these

Financial Planning Academy 19


Part 4

1 The bid-ask spread is best described by which one of the following statements?
(a) It is the brokers’ commission
(b) It is the dealers’ gross income from a transaction
(c) It is larger for illiquid securities than it is for liquid securities
(d) All of the above

2 How do T-Bills pay interest to their investors?


(a) Coupon interest
(b) Possible price appreciation above their discounted price
(c) T-Bills pay no interest
(d) Difference between issue price and face value

3 Which of the following are characteristics of Money Market Securities?


(a) They are issued by the Government , municipalities and large corporations that
have high –quality ratings.
(b) All have terms to maturity that are 270 days or less
(c) All lend to have large amounts of purchasing power risk
(d) Both (a) and (b)
(e) Both (b) and (c)

4 A basis point is which one of the following?


(a) One Rupee, Re.1
(b) One percentage point, 1percent
(c) One paisa, Re.0.01
(d) One one-hundredth of one percentage point, 0.01 of 1percent

5 Government Bond is best described by which one of the following statements?


(a) It has no voting privileges.
(b) It receives no cash dividends.
(c) It may be resold at any time.
(d) All of the above

6 Equity shareholders have which of the following rights?


(a) They can legally demand information from a corporation in which they are a
shareholder and thus gain access to its books.
(b) They can vote for the common shareholders’ dividend
(c) They can vote for the preference shareholders’ dividend.
(d) All of the above

Financial Planning Academy 20


7 Preference shareholders receive priority over common stockholders with respect to
which of the following?
(a) Dividends cannot be paid to common stockholders unless the preference stockholders
receive their stated dividend.
(b) In the event of bankruptcy and liquidation, the preference shareholders are paid
before the common shareholders.
(c) Preference shareholders get to elect the chairman of the corporation’s Board of
Directors.
(d) Both(a) and (b) are true, but (c) is false.
(e) All of the above are true

8 Which one of the following equations correctly defines the dividend yield (Y) from a
share of common stock?
(a) Y= (purchase price)+(cash dividend, if any)/purchase price.
(b) Y= (price change)+(cash dividend, if any)/purchase price.
(c) Y= price change/purchase price.
(d) Y= cash dividend (if any)/purchase price

9 A preference share is
(a) Pays fixed dividend
(b) A marketable security
(c) A debt security
(d) Both (a) and (b)
(e) All of the above

10 Which of the following statements best describes the convertibility of preference


shares?
(a) Some issues of preference share may be converted into common share at the option
of the investor any time and at a conversion ratio that never changes.
(b) Some issues of preference share may be converted into common share at the option
of the investor within a limited period number of years after the preferred stock is
issued.
(c) Some issues of preference shares may be converted into common share at the option
of the investor only after a specified number of years have elapsed since the
preference share was initially issued.
(d) All the above statements are true.
(e) Preference share is never a convertible security.

11 Which of the following statements best describe corporate bonds?


(a) Bond investors are creditors of the corporation.
(b) The majority of bonds make coupon interest payment once per annum.
(c) Both (a) and (b) are true
(d) None of the above are true

Financial Planning Academy 21


12 A Debenture Trust deed is best described by which one of the following statements?
(a) It is a legal contract describing the rights of specific bondholders.
(b) It describes the duties of the trustees.
(c) Both (a) and (b) are true
(d) None of the above is true

13 The quality ratings of a corporation’s bond issue are primarily determined by which of
the following?
(a) The issuer’s financial condition and the indenture contract that governs the
issuing firm.
(b) The calculations of ratios by the firms.
(c) The level of interest rates.
(d) The quality of management.

14 One bond with an AA-grade rating might pay a higher yield- to-maturity than another
AA-grade bond issued at a different time by the same corporation because of which of
the following reasons?
(a) Bonds with longer maturities always pay higher rates of interest than similar bonds
that have shorter maturities
(b) The bond market is sometimes irrational and evaluates the riskiness of some bond
issues erroneously.
(c) One bond issue is a secured one whereas other issue is unsecured.
(d) All of the above

15 Which of the following bond quality ratings applies to default –free-bonds?


(a) AAA
(b) AA
(c) Both (a) and (b) are default free.
(d) None of the above is default –free

16 A security will not earn the yield-to-maturity that was promised when the security
was purchased if which of the following conditions occurs?
(a) The issuer defaults on either the interest or principal payments.
(b) The investor sells the security prior to its maturity date.
(c) Cash flows from the security paid to the investor prior to its maturity date are held
in cash or spent on consumption goods rather than reinvested.
(d) All of the above are true
(e) None of the above is true

17 An investor that employed a naïve buy-and-hold strategy would be employing :


(a) An active portfolio management strategy
(b) A passive portfolio management strategy
(c) A balanced asset allocation
(d) None of these

Financial Planning Academy 22


18 All stock market indexes are most accurately characterized by which of the following
statements about the degree to which they covary together?
(a) They are perfectly positively correlated
(b) They are highly positively correlated
(c) They are uncorrelated
(d) They are negatively correlated
(e) It is impossible to generalize, some are highly positively correlated and some are
negatively correlate

In question number 19 to 22 state True or False


19 Depreciation must be omitted from a firm’s net profit in order to determine how much
cash flow the firm generated
(a) True
(b) False

20 When a firm pays creditors the transaction does not affect the equity capital shown in
its balance sheet in any way.
(a) True
(b) False

21 A primary issue of bonds or stock would increase both sides of the issuing company’s
balance sheet by the same amount.
(a) True
(b) False

22 The retention rate equals 100 percent less the percent of the corporation’s earnings
paid out for cash dividends.
(a) True
(b) False

23 Which of the following ratios will increase as a firm uses more financial leverage?
(a) The times-interest-earned ratio
(b) The debt to equity ratio
(c) The inventory turnover
(d) Both (a) and (b)
(e) Both(a) and (c)

24 Which of the following factors tends to increase the growth rate of a corporation?
(a) External borrowings
(b) Increasing the retention rate
(c) Increasing the rate of return on equity
(d) Both (a) and (b)
(e) All of the above

Financial Planning Academy 23


25 A company has total assets of 20,00,000. It has 7,00,000 in long-term debt. If the
stockholders’ equity is 9,00,000, what is its total debt /total assets ratio
(a) 45%
(b) 47%
(c) 59%
(d) 52%
(e) 55%
(HINT: TOTAL ASSETS=2000000 TOTAL DEBT =TOTAL ASSETS-STOCKHOLDERS’
EQUITY=1100000)

26 A company had a total debt to total assets ratio of. 0.4, total debt of Rs.2,00,000 and
net income of Rs.30000. Determine the corporation’s return on equity?
(a) 8%
(b) 9%
(c) 10%
(d) 12%
(e) 14%
(HINT: RATIO IS 0.4 MEANS TOTAL ASSETS ARE RS.500000 AND EQUITY PORTION IS
RS.3,00,000)

27 Assume the following information: stockholders’ equity=Rs.2000 shares


outstanding=40 market price to book value =2. Determine the market price for the firm’s
common stock?
(a) Rs.75
(b) Rs.100
(c) Rs.110
(d) Rs.115
(e) Rs.117
(HINT : 2000/40=50 IS THE BOOK VALUE. MP TO BOOK VALUE IS=2 THEREFORE MP IS 100)

28 Which of the following is a source of funds?


(a) An increase in inventory
(b) An increase in accounts receivable
(c) An increase in investments
(d) An increase in accounts payable
(e) None of these

29 Assume the correlation coefficient r between the rates of return from these two
automobile sector shares, say, A and B was +0.8. If you took a long position in A and
short position in B (or vice versa) of exactly equal value you would be perfectly hedged.
(a) True
(b) False
(HINT : IF CORRELATION COEFFICIENT IS 0.8 THEN FOR HEDGING (RS.100000) ONLY 80000 IS
TO BE INVESTED)

Financial Planning Academy 24


30 Selling securities short is useful in which of the following statements?
(a) Speculating
(b) Hedging
(c) Arbitrage
(d) All of the above

31 Why do share prices usually drop when news about decline in a company’s earnings
per share is reported?
(a) Because a reduction in a earnings means that the firm has less money with which to
pay dividends and therefore the market fears a reduction in the company’s future
dividends.
(b) Because the share market anticipates that a decreased level of earning power might be
the indicator of default and perhaps even bankruptcy.
(c) The statement is false. Share prices do not usually react about current earnings.
(d) Both (a) and (b) are true

32 Interest-rate risk is defined by which of the following statements?


(a) Fluctuations in the coupon interest rates from one bond issue to another.
(b) Fluctuations in the market prices of bonds as their prices move inversely to the
prevailing market interest rates.
(c) The variability of returns as a result of fluctuations in market interest rates.
(d) Both (a) and (b)
(e) All of the above

33 Calculate the (i) expected rate of return for M/S X Ltd. from the following information

Rates of return (%) Probability


-0.10 0.10
-0.20 0.25
0.20 0.30
0.50 0.25
0.30 0.10
(a) 16.5%
(b) 15.5%
(c) 14.0%
(d) 16.5%

34 Which of the following describe “Risk”?


(a) Risk means possibility of expected return deviating from actual return.
(b) Bond quality ratings also describe the level of default risk.
(c) T-Bills are called risk free but they also contain some amount of interest rate risk.
(d) Both (b) and (c) are true
(e) All the above are true

Financial Planning Academy 25


35 XYZ ‘s price is Rs.35 per share and its expected dividend is Rs.4.20 per share, which
is growing at the rate of 8% per annum. Determine the Required rate of return?
(a) 20%
(b) 18%
(c) 19%
(d) 19.5%

36 If ABC pays dividend of Rs.3.85 per share which is growing at a 7 percent rate per
year and is expected to grow at the same rate in future. Its required rate of return is
14.5%. Determine its share value.
(a) 52.48
(b) 49.25
(c) 54.93
(d) 55.75
(e) 47.26
(3.85 is current year’s dividend. D1 will be 3.85*1.07=4.1195)

37 A company has current earnings per share of Rs.7. Assume a dividend payout of
60%.Earnings grow at a rate of 8% per year If the required rate of return is 15 percent,
what is the current share value?
(a) 64.80
(b) 65.20
(c) 63.56
(d) 68.35

38 The value of an option tends to increase as the volatility (or risk) of the underlying
asset increases:
(a) True
(b) False

39 If you purchase a put option, you are expecting the value of the underlying asset to :
(a) Increase
(b) Decrease

Use the information given below to answer the questions (40-46)


Share Current Exercise Time to
price price maturity
Call Call Put Put
Premium premium premium premium
3 months 6 months 3 months 6 months
A 52 50 3 4 0.35 1.05
B 40 45 1 1.25 5.5 6.00
C 35 30 6 6.3 0.45 0.65

Each contract is equal to 100 shares.

Financial Planning Academy 26


40 If you purchase one 3-month call contract on A, what profit or loss will you make at
the maturity date if the price of A at that time is Rs.57?
(a) Rs.200
(b) Rs.400
(c) Rs.460
(d) Rs.500
(e) Rs.560

41 If B’s price is Rs.35 at the maturity of the 6-month options, determine the value of
five 6-month put contracts at their maturity date?
(a) Rs.2000
(b) Rs.5700
(c) Rs.8200
(d) Rs.4000
(e) Rs.3600

42 If you had purchased five 3-month call options of C and the price of C’s share is
Rs.32 at maturity. Determine your profit or loss on the investment.
(a) Rs.1000
(b) Rs.1500
(c) Rs.2000
(d) Rs.4000
(e) Rs.500

43 If you had purchased five 3-month puts on C, what would your profit or loss position
have been on maturity if the share’s price were Rs.32?
(a) -Rs.225
(b) –Rs.400
(c) –Rs.600
(d) Rs.400
(e) Rs.600

44 Your client wrote five 6-month call options on B’s share. What is his profit or loss
on the options at maturity if the price of B at that time is Rs.43?
(a) Rs.625
(b) Rs.600
(c) Rs.400
(d) Rs.300
(e) Rs.200

Financial Planning Academy 27


45 If your client had written five 6 Month put option on B, what would his profit or loss
have been at the maturity of the options if the share price was Rs.43 per share?
(a) Rs.1000
(b) Rs.2000
(c) Rs.1800
(d) Rs.1500
(e) Rs.500

46 Which of the following options are in the money?


(a) A’s 3-month call
(b) B’s 6 month put
(c) C’s 6-month put
(d) (a) And (b)
(e) None of the above

47 If an investor is bearish on a share, buying a put option is usually better than selling
short because
(a) The holders’ losses can be no more than the put premium if the share price rises, but
the short seller’s losses could be unlimited in this situation.
(b) The short sale will become worthless after a short period of time but the put option
will not become worthless.
(c) The short seller must pay any dividends paid by the security the short seller
borrowed.
(d) Both (a) and (c)
(e) Both (a) and (b)

48 The Black Scholes model cannot be used to determine the overall market value of a
firm.
(a) True
(b) False

The returns of 2 shares are as follows


Year Returns X Returns Y
2001 20% 30%
2002 -10% -20%
2003 15% 18%
2004 17% 10%
2005 19% 5%
49 Calculate the covariance of returns.
(a) 220.32
(b) –420.11
(c) 145.22
(d) 270.36
(e) 162.08
(HINT: COV(XY)={(RIX-E(R X) (RIY-E(RY )}

Financial Planning Academy 28


50 Calculate the correlation coefficient from the above information.
(a) .543
(b) .925
(c) .869
(d) .785
(e) .695
(HINT: RXY=COV XY/SD X SD Y)

51 Duration of a zero coupon bond is


(a) Equal to its term to maturity
(b) Less than its term to maturity
(c) More than its term to maturity
(d) None of these.

52 What is true about the characteristics of Bond?


(a) Long-term bonds are almost always more volatile in terms of price than short-term
bonds for a given change in interest rates
(b) Short term bonds are less vulnerable to interest rates fluctuation than long term bonds
(c) both (a) and (b) are true
(d) Both statements are false

53 Duration for a coupon-paying bond is always less than its term to maturity.
(a) True
(b) False

54 Compute the price of a bond when following information is given?


Coupon – 10%semi annually
YTM –12%
Term to maturity –10 years
(a) Rs.1051.65
(b) Rs.1159.88
(c) Rs.885.30
(d) Rs.955.41
(e) Rs.889.88

55 The Price of the bond in above question after 2 years, assuming everything else
remains the same, is
(a) Rs.1130.55
(b) Rs.935
(c) Rs.780
(d) Rs.860
(e) Rs.898.94
(HINT: NUMBER OF YEARS WILL BE 8 INSTEAD OF 10 AS N= NUMBER OF YEARS TO
MATURITY)

Financial Planning Academy 29


56 A bond’s duration measures which one of the following?
(a) The time structure of a bond’s cash flows
(b) The bond’s interest rate risk
(c) Both (a) and (b) above
(d) The default risk of the bond issue
(e) None of the above

57 If the market rate of interest falls, a coupon –paying bond will


(a) Decrease in value
(b) Experience a decrease in duration
(c) Experience an increase in duration
(d) None of these
(e) Both a and b above

58 A bond’s reinvestment rate risk:


(a) Refers to the problem of being able to purchase another bond with the same or higher
YTM when the existing bond matures or is called
(b) Is the risk of not being able to reinvest the coupons of a bond at the bond’s YTM
(c) Is the same as marketability risk
(d) None of these

59 If there is an expectation of large decline in interest rates, which of the following


investments should you choose?
(a) Money market fund
(b) Low-coupon short term bond
(c) High coupon short term bond
(d) Long term zero coupon bond
(e) Short term zero coupon bond

60 Bonds with higher coupons, other things being the same


(a) Have more interest rate risk than bonds with smaller coupons
(b) Have less interest rate risk than bonds with smaller coupons
(c) Have higher duration than small coupon bonds
(d) Have lower duration than small coupon bonds
(e) Both b and d

61 Convertible debentures are valued on the basis of:


(a) Yield to maturity
(b) Market value
(c) Cost basis
(d) Convertible portion like equity instruments and non-convertible portion as debt
instrument

Financial Planning Academy 30


62 The facility given by Mutual Fund to the investors wherein they can transfer on
periodic basis a specified amount from one scheme to another within a same fund family
is called?
(a) Systematic withdrawal plan
(b) Systematic investment plan
(c) Automatic reinvestment plan
(d) Systematic transfer plan

63 Are Mutual funds allowed to participate in security lending?


(a) Yes
(b) No

64 Which of the following statements is false?


(a) Mutual Funds are allowed to invest in overseas securities, subject to the approval of
Board, without any investment cap or ceiling.
(b) Mutual Funds are allowed to trade in derivatives for the purpose of hedging, portfolio
balancing and speculation.
(c) Both the statements are false.
(d) Both are true

65 Mutual Funds can borrow money for a period


(a) Not exceeding one year, upto25% of net assets of the scheme.
(b) Not exceeding 3 months, upto20% of net assets of the scheme
(c) Not exceeding six months, upto20% of the net assets of the scheme.
(d) For ant period, no limit

66 A scheme of mutual fund may be wound up, after repaying he amount due to the unit
holders.
(a) On the happening of any event which in the opinion of the trustees requires the
scheme to be wound up
(b) If 75% of the unit holders of a scheme pass a resolution that the scheme to be wound
up
(c) If the Board so directs in the interest of the unit holders
(d) Any of the above

67 Mutual funds can guarantee return on any of its scheme if


(a) The asset management company fully guarantee the return
(b) The sponsor fully guarantee the return
(c) Both (a) and (b) above
(d) Mutual fund cannot guarantee any return

Following 2 questions relate to RBI savings bond.


68 Premature withdrawals is not allowed
(a) True
(b) False

Financial Planning Academy 31


69 Non resident Indians are allowed to invest in the bonds
(a) True
(b) False

70 There is lock in period?


(a) True
(b) False

71 Which of the following instruments are useful in managing interest rate risk?
(a) Forward rate agreements
(b) Interest rate swaps
(c) Commercial papers
(d) Both (a) and (b)

72 Repurchase agreements are?


(a) Secured loans
(b) Unsecured loans
(c) Long term investment
(d) Carries no interest

73 Treasury bills are?


(a) Rediscounted by the RBI
(b) Discount securities
(c) Issued to meet the short term needs of the government
(d) All of the above

74 The norms for CP issue is prescribed by


(a) Issuing authority
(b) SEBI
(c) RBI
(d) Banks

75 Due to changes in interest rates, there may be a situation where there is a price gap
between new and existing securities traded in the market place. When the yield exceeds
the return from the coupon payment alone , the security is said to be traded at
(a) Par
(b) Discount
(c) Premium
(d) None of these

76 Protective (call or put) option means


(a) Holding only call or put options
(b) Having the securities and no option
(c) Holding securities and writing the call
(d) Holding securities and writing the put

Financial Planning Academy 32


77 Naked positions in the derivatives market refers to
(a) No derivative instruments in portfolio
(b) Having the securities and equivalent derivative instruments
(c) Holding securities and buying the call
(d) Only call or put or futures open position

78 Which of the following legislations are applicable to Mutual Funds in India?


(a) Companies Act, 1956
(b) SEBI Act, 1992
(c) Indian Trust Act, 1882
(d) All of the above

79 The custodian is appointed by


(a) Asset Management Company
(b) SEBI
(c) Board of Trustees
(d) Sponsors

80 What is the contribution of sponsor to the net worth of asset management company?
(a) Maximum 40%
(b) Minimum 40%
(c) Maximum 75%
(d) Minimum 25%

81 Companies Act lays down rules and regulations regarding


(a) Issue, allotment and transfer of securities
(b) Standard of disclosure in public issue
(c) Underwriting of issue
(d) All of the above

82 Shareholders in a limited company are not liable for


(a) The paid amount of capital in that company
(b) The assets of the limited company
(c) The debts of the company if his shares are fully paid up
(d) The goodwill of the company.

Covered call writing is a strategy preferred by risk-averse investors. Consider an investor


who writes a covered call on ABC share. Spot price is 38,Exercise price is 40 and 3-
month call on ABC share is traded at 3.

83 What is the initial cash flow incurred at the time of investment?


(a) 38
(b) 35
(c) 41
(d) 37

Financial Planning Academy 33


84 What is the maximum profit realized from this strategy?
(a) 2
(b) 4
(c) 5
(d) 0

85 A speculator is a trader who


(a) Assumes no risk for profit
(b) Assumes risk for profit
(c) Does not incur losses at all
(d) Has a long position in cash market and long position in derivative market.

Consider two stocks P and Q


Expected return Standard deviation
Stock P 16% 25%
Stock Q 18% 30%

86 What is the weight that drives the standard deviation of portfolio to zero when the
returns are perfectly negatively correlated?
(a) Wp=0.545,Wq=0.455
(b) Wp=0.640, Wq-0.360
(c) Wp=1.000,Wq=0.000
(d) None of these.
(hint: Wp=SD of Q/(Sd of P+ Sd of Q)=30/25+30=0.545)
{wq (weight of Q)=Sd of Q/SD of p+ Sd of Q=25/30+25=0.455)

87 What is the expected return of a portfolio constructed to drive the standard deviation
of portfolio return to zero when the returns on the two stocks are perfectly negatively
correlated?
(a) 15.75%
(b) 16.91%
(c) 16.50%
(d) 17.25%

88 What is Market Risk Premium?


(a) It is premium available in the market.
(b) It is the market risk measured by premium paid
(c) It is the difference between the average return on the portfolio and average risk
free rate.
(d) Any of these

Financial Planning Academy 34


89 Total risks comprises of:
(a) Market risk
(b) Systematic risk +unsystematic risk
(c) Political risk
(d) Inflation risk

90 Which of the above mentioned could be reduced after appropriate diversification?


(a) Market risk (systematic risk)
(b) Unsystematic risk (company specific risk)
(c) Political risk
(d) Inflation risk

91 What is Call Option in debt securities?


(a) It is the risk that the bond may be called before maturity.
(b) It is available to the issuer only.
(c) Issuer will call the bond when interest rates have fallen
(d) All of these.

92 The important vectors of an active strategy are?


(a) Market timing
(b) Sector rotation
(c) Security selection
(d) Use of specialized investment concepts
(e) All of these
(f) a b and c

93 Investors who subscribe to the view that the market is efficient, adopt which of the
following strategy?
(a) Active strategy
(b) Passive strategy
(c) Market timing strategy
(d) Buy and sell strategy

94 An Active Fund management strategy is followed by?


(a) Investment professionals
(b) Fund managers
(c) Businessmen
(d) A b and c
(e) A and b

Financial Planning Academy 35


95 What is true about Fundamental Analysis?
A. It involves research into the operations and finances of a company
B. It involves estimating the future earnings of a company
C. It involves study of price behavior of a company’s stocks.
(a) (a) and (b)
(b) (a) (b) and (c)

96 Hedger is a trader
(a) Who enters derivative market in order to reduce a pre existing risk
(b) Who is a trader who enters derivative market in anticipation of a need in the near
future
(c) Whose net wealth change at the time the derivative contract expires is expected to be
zero (perfect hedge assumed)
(d) All of the above

97 The present value of a sum of money---------as the Discounting rate ---------


(a) Remains same, Increases
(b) Decreases, Decreases
(c) Increases, Increases
(d) Increases, Decreases
(e) Data insufficient

98 A growth oriented non dividend paying share is bought for Rs.250 and sold Rs.450
after 5 years, the compound annual growth rate is:
(a) 14.86%
(b) 12.47%
(c) 11.50%
(d) 10.71%
(hint : PV= -250, FV=450, 5=N Comp I)

99 The price of Stellar Ltd is currently Rs.40. The dividend next year is expected to be
Rs.4.00. Required return on stocks is 12%. Find the expected growth rate under the
Constant Growth Model.
(a) 2.00%
(b) 2.25%
(c) 1.90%
(d) 2.75%
(hint: Po=40 D1=4.00 R=12% find g(growth rate)

100 A 5 year annual annuity has a yield of 6%. What is the duration?
(a) 2.88 years
(b) 2.55 years
(c) 3.16 years
(d) 1.35 years
(hint: formula {(1+yield)/yield- number of payments/ (1+yield)^ number of payments-1
(Formula given in Prasanna chandra book in bond valuation)

Financial Planning Academy 36


Part 5

Note: S = Sigma in Greek Alphabet

1. The unique risk of a security is diversifiable.


(a) True
(b) False

2. Market Risk can also be called Unsystematic risk.


(a) True
(b) False

3. ABC Company paid a dividend of Rs.5.40 during the year 2005. Amit had bought a
share at Rs.61.20 at the beginning of the year. ABC’s price at the end of the year is
Rs.72.40. Find the total return on the stock.
(a) 25.54%
(b) 8.85%
(c) 27.54%
(d) 19.20%

4. During the last 10 days, a share was traded at the following rates. 1st day – Rs.152,
2nd day – Rs.145, 3rd day – Rs.150, 4th day-Rs.151, 5th day - Rs.147,
6th day – Rs.155, 7th day – Rs.153, 8th day – Rs.149, 9th day – Rs.148 &
10th day – Rs.150.
Find the standard deviation of the share.
(a) 8.66
(b) 2. 9
(c) 3. 5
(d) 4.33

5. A stock caries the following returns over a five year period: R1 = 0.20, R2 = 0.10,
R3 = 0.18, R4 = 0.12, R5 = 0.16. Calculate the following:
A. Arithmetic mean return, B. Cumulative wealth index & C. Geometric mean return.
(a) 11.5%, 1.4557, 18.61%
(b) 11.2%, 1.5557, 10.25%
(c) 11.2%, 1.3457, 10.51%
(d) 11.2%, 1.6557, 10.61%

6. What is the standard deviation of the stock described above?


(a) 149.25
(b) 10. 5
(c) 12. 2
(d) 11. 5

Financial Planning Academy 37


7. The probability distribution of the rate of return on ABC stock is given below:

State of Economy Probability of Occurrence Rate of Return


Boom 0.40 25%
Normal 0.30 12%
Recession 0.30 - 6%

What is the standard deviation of return?


(a) 11.4%
(b) 12.9%
(c) 10.5%
(d) 13.5%

8. You are thinking of buying some shares of XYZ Ltd. The rates of return expected is
as follows:

Possible Rate of Return Probability


.07 .10
.10 .50
.11 .20
.09 .10
Compute the expected return on the investment.
(a) 7.8%
(b) 8.8%
(c) 5.5%
(d) None of the above

9. The expected return and standard deviations of stock A & B are:

Stock Expected Return Standard Deviation

A 15% 10%
B 5% 18%

Amit buys Rs.20,000 of Stock A and sells short Rs.10,000 of Stock B using all the
Proceeds to buy more or Stock A. The correlation between the two securities is .35.
What are the expected return & standard deviation of Amit’s portfolio?
(a) 3.5%, 15.5%
(b) 8.8%, 7.03%
(c) 20% , 14.5%
(d) 9.8%, 15.6%

Financial Planning Academy 38


10. Ashish owns three stocks & has estimated the following joint probability distribution
of returns:

Outcome Stock X Stock Y Stock Z Probability


1 0 5 0 .20
2 -10 10 5 .10
3 20 10 10 .50
4 10 -20 15 .20

Calculate the portfolio’s expected return & standard deviation if Ashish invests 20% in
stock X, 50% in stock Y & 30% in stock Z. Assume that each security is completely
uncorrelated with the return of other securities.
(a) 6.25%, 6.54
(b) 5.25%, 2.55
(c) 8.45%, 3.75
(d) 4.25%, 6.54

11. Stock A & B are positively correlated with a correlation co efficient of .75. When
stock A moves up by 12%, how will stock B perform?
(a) Stock B will move up by 12%
(b) Stock B will move down by 12%
(c) Stock B will move up by 9%
(d) Stock B will move down by 9%

12. Beta captures all the inherent risk in an individual security.


(a) True
(b) False

13. Unsystematic risk is reduced in a portfolio because securities are not perfectly
correlated.
(a) True
(b) False

14. As Beta increases, the expected return also increases.


(a) True
(b) False

15. Rational investors will form portfolios to eliminate systematic risk.


(a) True
(b) False

Financial Planning Academy 39


16. For calculating portfolio risk, we need information of which of the following things?
(a) The weights of each security in the portfolios.
(b) The co-movements between the different securities measured by co-variance.
(c) Both A and B above
(d) None of the above

17. As the risk in a portfolio is largely determined by co-variance of different securities in


the portfolio, the portfolio risk can be reduced when the securities are less than
perfectly positively correlated.
(a) True
(b) False

18. Consider 2 stocks A and B

Expected Return Standard Deviation


Stock A 14% 22%
Stock B 20% 35%

The returns on the stocks are perfectly negatively correlated. What is the expected return
of a portfolio above securities of A & B when the portfolio is so constructed so as to
make the standard deviation of portfolio return to zero?
(a) 15.25%
(b) 16.31%
(c) 12.45%
(d) None of the above

19. The following information is available.

Stock A Stock B
Expected Return 15% 14%
Standard Deviation 15% 8%
Co efficient of correlation: .60
What is the covariance between Stocks A & B?
(a) 72
(b) 65
(c) 70
(d) None of the above

Financial Planning Academy 40


20. A portfolio consists of 3 securities. The proportion of these 3 securities are:
W1 = .2, W2 = .4, W3 = .4
The standard deviation of return on these securities in terms of percentage is:
W1 = 6, W2 = 9, W3 = 10

The correlations efficient among security returns are:


P12 = .4, P13 = .6, P23 = .4

What is the standard deviation of the portfolio?


(a) 2.5
(b) 3.5
(c) 7.7
(d) 6.9

21. About 85% to 95% of the Portfolio return comes from:


(a) Asset Allocation Policy Decision
(b) Appropriate Timing
(c) Individual Security Selection
(d) B and C above

22. An investment can be termed as short-term investment or a long-term investment


based on the timing the investor is expected to hold the investment.
(a) True
(b) False

Read the data below and answer the questions below:


Risk free Rate of Return : 7%
Expected Return on Market Portfolio : 15%
Beta of security X : .7
Beta of security Y : 1.5

23. Expected Return on Security X is:


(a) 12.2%
(b) 12.6%
(c) 10.5%
(d) None of the above

24. Expected Return on Security Y is:


(a) 15.4%
(b) 12.0%
(c) 19.0%
(d) None of the above

Financial Planning Academy 41


25. If expected return on security X is 20%, its Beta will be:
(a) 1.62
(b) 2.04
(c) 1.34
(d) None of the above

26. If the risk free rate is 6% and the expected return is 20%, beta of security X will be:
(a) .25
(b) 1.35
(c) 1.55
(d) None of the above

27. CAPM helps us to know whether the expected return from a security is in line with its
fair return.
(a) True
(b) False

Read the data given below & answer the following questions:

Standard deviation of an asset : 2.5%


Market Standard Deviation : 2.0%
Risk free Rate of Return : 13.0%
Expected Return on Market Portfolio : 15.0
Correlation co-efficient of portfolio with market : 0.8

28. Market sensitive index will be:


(a) 1.5%
(b) .8%
(c) 10
(d) 1

29. Expected Return on the Portfolio will be:


(a) 13%
(b) 15%
(c) 14%
(d) 16%

30. If portfolio beta is .5 and risk free return is 10%, expected return on the portfolio will
be:
(a) 10%
(b) 15%
(c) 12.5%
(d) 11.25%

Financial Planning Academy 42


Answers

1. True

2. False

3. C Total Return = Dividend Paid + (Ending Price – Beginning Price)


Beginning Price

= 5.40 + 72.40 - 61
61

= 27.54%

4. B

Day Rate Deviation Square of Deviation


(Ri – R) (Ri –R)2

1 152 152-150 4
2 145 145-150 25
3 150 150-150 0
4 151 151-150 1
5 147 147-150 9
6 155 155-150 25
7 153 153-150 9
8 149 149-150 1
9 148 148-150 4
10 150 150-150 0

(S)2 = ( { (Ri – R)2 ) = 78 = 8.66


N-1 9

S = Square root of variance

= Square root of 8.66 = 2.9

5. D
a) Arithmetic mean
R = .20 - .10 + .18 + .12 +.16
5

= .112 or 11.2%

Financial Planning Academy 43


b) Cumulative Wealth Index

CW15 = 1 (1.20 + .90 +1.18 + 1.12 +1.16)

= 1.6557

c) Geometric Mean Return

GM = (1.20 + .90 + 1.18 + 1.12 + 1.16) 1/5 - 1


= 10.61%

6. c)

7. b)

8. b) Expected Return = (.07 * .10) + (.10 * .50) + (.11 * .20) + (.09 * .10)

= 8.8%

9. c) Expected Return = (1.5 * 15%) + (-.5 * 5%) = 20%

(S) P = [(1.5)2 * (10)2] + (-.5)2 * (18)2 +


[(2* 1.5* -.5* .35* 10*18)]1/2

= (211.5)1/2 = 14.5%

10. a) The expected return on three securities of Ashish’s portfolio:


Return on X (.20* 0%) + (.10* -10%) + (.50* 20%) + (.20* 10%) = 11%
Return on Y (.20* 5%) + (.10 *10%) + (.50* 10%) + (.20* 20%) = 3
Return on Z (.20* 0%) + (.10* 5%) + (.50* 10%) + (.20*15%) = 8.5%

Expected Return on Portfolios :

(.20* 11) + (.50*3) + (.30* 8.5) = 6.25%

The S of the three securities are :

S X = [ {.20 * (-5)2 } + {.10* (-15)2} + {.50* (15)2} + {.20* (5)2]1/2

= (5 + 22.5 +112.5 + 5)1/2

= (145)1/2 = 12.04

S Y = [ {.20* (3.75)2} + {.10* (8.75)2} + {.50* (8.75)2} +

Financial Planning Academy 44


{.20* (-21.25)2]1/2

= (2.8 + 7.6 + 38.3 + 90.3)1/2

= (139)1/2 = 11.78

S Z = [ {.20* (-7.5)2} + {.10* (-2.5)2 } + {.50* (2.5)2} +


{.20*(7.5)]1/2

= (11.25 + .62 + 3.1 + 11.25)1/2


= (26.22)1/2 = 5.12

S P = [ {(.20)2* (12.04)2} + {(.50)2 * (11.78)2} + {(.30)2* (5.12)2 ] ½


= 6.54

11. c)

12. False

13. True

14. True

15. False

16. c)

17. True

18. b) The weights that drive the standard deviation of the portfolio to zero,
when the returns are perfectly negatively correlated are:
WA = SB
SA +SB

= 35 = .614
22 + 35

Expected return of the portfolio is : (4 * .614 + 20* .386)


=16.31%

19. a)

Financial Planning Academy 45


20. c) SP = [ { (.2)2 * (6)2 + (.4)2* (9)2 +(.4)2* (10)2 + 2* .2 *.4* .4* 6*9
+2* .2* .4* .6* 6* 10 + 2* .4* .4* .7* 9* 10]1/2

= (1.44 + 12.96 +16 + 3.46 + 5.76 + 20.16)1/2

= (59.78)1/2 = 7.7

21. a)

22. True

23. b)

24. c)

25. a)

26. c)

27. True

28. D

29. B

30. C

Part 6

Financial Planning Academy 46


Portfolio Rebalancing means reviewing and revising the portfolio composition i.e. stock
bond mix.

The three policies with respect to portfolio rebalancing are:

Buy and Hold Policy also called Drifting Asset Allocation


Constant Mix Policy also called Balanced Asset Allocation
Constant Proportion Portfolio Insurance (CPPI) also called Dynamic Asset Allocation.

(a) It calls for increasing exposure to stocks when the portfolio appreciates in value
(a) Buy & Hold Policy
(b) Constant Mix Policy
(c) CPPI Policy
(d) None of the above

(b) It calls for maintaining the proportion of stocks and bonds in line with their target
value
(a) Drifting Asset Allocation
(b) Balanced Asset Allocation
(c) Dynamic Asset Allocation
(d) None of these

(c) It calls for selling stocks as they fall and buying stocks as they rise
(a) Buy & Hold Policy
(b) Constant Mix Policy
(c) CPPI Policy
(d) None of these

(d) It performs well in flat but fluctuating market


(a) Drifting Asset Allocation
(b) Balanced Asset Allocation
(c) Dynamic Asset Allocation
(d) None of these

5.Show the pay off from an initial investment of 1,00,000 when the market moves from
100 to 70 and back to 120 under the following policies:
(a) Drifting Asset Allocation policy under which the stock bond mix is 60 : 40
(b) Balanced Asset Allocation policy under which the stock bond mix is 60 : 40
(c) CPPI Policy which takes the form: Investment in stock = 3
(Portfolio Value – 80000)

6. Which of the following asset allocation would you recommend to as 60-year-old


retiree who depends on his investments for his monthly income?

Financial Planning Academy 47


(a) Fixed Deposits 60% Properties 30% Equities 10%
(b) Fixed Deposits 40% Properties 30% Equities 30%
(c) Fixed Deposits 20% Properties 40% Equities 40%
(d) Fixed Deposits 10% Properties 40% Equities 50%

7. The standard deviation of the returns of a portfolio of securities will be _________ the
weighted average of the standard deviation of returns of the individual component
securities.
(a) Equal to
(b) Less than
(c) Greater than
(d) Less than or equal to (depending upon the correlation between securities)

8. Modern “ Asset Allocation” is based upon the model developed by Harry Markowitz.
Which of the following statement is/are correctly identified with this Model.
(i) The risk, return and co-variance of assets are important input variables in creating
portfolios.
(ii) Negatively correlated assets are necessary to reduce the risk of portfolios.
(iii) In creating a portfolio, diversifying across asset types e.g. (stocks and bonds) is
less effective than diversifying within an asset type.
(iv) The effective frontier is relatively insensitive to input variable

(a) 1 & 2 only


(b) 1, 2 & 3 only
(c) 1 only
(d) 1, 2 & 4 only

9. Given the following diversified mutual fund performance data, which fund has the best
risk adjusted performance if the risk free rate of return is 5.7%

Fund Average Standard Deviation Beta


Annual Return of Annual Return
a .0782 .0760 .950
b .1287 .1575 1.250
c .1034 .1874 .857
d .0750 .0810 .300

(a) Fund b because the annual return is highest


(b) Fund a because the Treynor ratio is lowest
(c) Fund d because the Treynor ratio is highest
(d) Fund c because the Sharpe ratio is lowest

10. Which of the following investment vehicles are most appropriate for an emergency
fund for a family with Rs. 12000/- monthly income

Financial Planning Academy 48


(a) Balanced Mutual Funds
(b) Growth Funds
(c) Income Funds
(d) Money Market Mutual Funds

11. Which of the following statements about investment risk is correct?


(i) Beta is a measure of systematic, non diversifiable risk
(ii) Rational investors will form portfolios and eliminate systematic risk
(iii) Rational investors will form portfolios and eliminate unsystematic risk
(iv) Systematic risk is the relevant risk for a well-diversified portfolio
(v) Beta captures all the risk inherent in individual security

(a) 1, 2 and 5 only


(b) 1, 3 and 4 only
(c) 2 and 5 only
(d) 2, 3 and 4 only

12. Which combination of the following statements is true regarding the investment
strategy known as “dollar-cost averaging”?
(i) Invests the same dollar amount each month over a period of time.
(ii) Purchase the same number of shares each month over a period of time.
(iii) Lowers average cost per share over a period of time.
(iv) Invests the same dollar amount each month to protect the investment from loss
of capital.

(a) 1 and 2 only ( c) 2 and 3 only


(b) 1 and 3 only (d )1, 2 and 3 only

13. You have to decide between Performance Fund and Exit Fund. The risk free rate is
8%. Average return on Performance Fund is 18% and Exit Fund is 16%. The standard
deviation is 20% and 15% respectively and Beta of the two funds is 1 and 1.5
respectively. What is the Sharpe performance measure for Exit fund?
(a) .433
(b) .533
(c) .563
(d) .536

14. What is Sharpe performance for Performance Fund


(a) .345
(b) .600
(c) .500
(d) .545

Answers

Financial Planning Academy 49


1. C

2. B

3. C

4. B

5. Market level is 100

Portfolio

Stocks Bonds Total

Buy & Hold Policy 60,000 40,000 1,00,000


Constant Mix Policy 60,000 40,000 1,00,000
CPPI Policy 60,000 40,000 1,00,000

Market falls to 70

Portfolio before Rebalancing Portfolio after Rebalancing

Stock Bond Total Stock Bond Total

Buy & Hold Policy 42000 40000 82000 42000 40000 82000
Constant Mix Policy 42000 40000 82000 49200 32800 82000
CPPI Policy 42000 40000 82000 6000 76000 82000

Market rises to 120

Buy & Hold Policy 71996 40000 111996 71996 40000 111996
Constant Mix Policy 84339 32800 117139 70283 46856 117139
CPPI Policy 10285 76000 86285 18855 67430 86285

6. A – A retiree at 60 has little appetite for risk as he is no longer has any earning power.

Financial Planning Academy 50


But a small amount of equity is still recommended as a hedge against inflation.

7. D

8. C

9. C

10. D

11. B

12. B

13. B

14. C

Part 7

Financial Planning Academy 51


1.Using the Gordon Constant Growth Model, Calculate the price of stock of company A,
with the following information

EPS = Rs. 10 per share


Current Dividend = Rs. 8 per share
Dividend Growth rate 5%
Risk free rate = 6%
Risk Premium = 7%

(a) Rs. 115/-


(b) Rs. 105/-
(c) Rs. 100/-
(d) None of the above

2. Which of the following have a negative impact on stock prices:


(i) Increase in Risk Premiums
(ii) Increase in dividend Growth Rate
(iii) Increase in Dividend Rate
(iv) Increase in Interest Rate

(a) 1, 2 & 3 only


(b) 1, 2 & 4 only
(c) 1, 3 & 4 only
(d) 2, 3 & 4 only

3. Company ABC is currently trading at Rs. 35 and pays a dividend of Rs. 2.30.
Analysts’ project a growth rate of 4%. Your client requires a rate of 9% to meet his stated
goal and wants to know whether he should purchase stock in company ABC.
(a) Yes, the stock is undervalued
(b) No, the stock is overvalued
(c) Yes, the required rate is higher than the projected growth rate.
(d) No, the required rate is lower than the expected rate.

4. “Stock prices adjust rapidly to the release of all new public information”. This
statement is an expression of which of the following ideas:
(a) Random Walk Hypothesis
(b) Arbitrage Pricing theory
(c) Semi-strong form of the efficient market hypothesis
(d) Technical Analysis

5. The Zeta Corporation’s current dividend is Rs. 3.85. If future dividends are expected to
grow at 4% forever, which of the following amounts should Zeta stock sell for if the
required rate of return on the stock is 14%.

Financial Planning Academy 52


(a) 28.57
(b) 38.50
(c) 40.04
(d) 41.60

6. The equity share of Abhishek Industries is currently selling at Rs. 25 per share. The
dividend expected next year is Rs. 2.50. The investors required rate of return on the stock
is 15 percent. If the constant growth model applies to Abhishek Industries, what is the
expected growth rate?
(a) 15%
(b) 5%
(c) 11%
(d) None of the above.

7. Fizzle Ltd. is facing gloomy prospects. The earnings and dividend are likely to decline
by 4%. The previous dividend was Rs. 1.50. If the current market price is Rs. 8.00, what
rate of return do investors expect from the stock?
(a) 18%
(b) 22%
(c) 14%
(d) None of the above.

8. A share’s earnings and dividends have been growing @18% per annum. The growth
rate is expected to continue for 4 years. After that the growth rate will fall to 12% for the
next 4 years. Thereafter the growth rate is expected to be 6% forever. If the last dividend
per share was Rs. 2.00 and the investors required rate of return on the stock is 15 %, what
is the intrinsic value per share?
(a) 16.83
(b) 23.49
(c) 40.32
(d) None

9. A firm has expected dividend pay out ratio of 50%, a required rate of return of 12%
and an expected growth rate of 5%. What is the firm’s expected P/E ratio? If the investor
expects next year’s earnings to be Rs. 5/-, what is the value of the stock today?
(a) 35.70
(b) 7.14
(c) 30.15
(d) None of the above

10. What is the value of the stock of Abhishek industries if it paid a dividend of Rs. 3 last
year and dividends are expected to grow @5% forever. The risk free rate is 8% and
the expected return on the market is 15% and the stock’s beta is 1.5.

Financial Planning Academy 53


(a) 20.23
(b) 22.15
(c) 15.28
(d) 23.33

11.Hindustan Ltd. has issued a preferred stock that pays a dividend of Rs. 20 per share.
The dividend is fixed and share has no expiration date. What is the intrinsic value of
Hindustan Ltd. preference share at a discount rate of 12%.
(a) 166.67
(b) 160.67
(c) 116.67
(d) None of the above.

12. You have invested in 4 securities. Company has an expected return of 15%. Company
B has an expected return of 17%. Company C has an expected return of 12% and
company D has an expected return of 10%. You have invested Rs. 40,000. What more
information is needed to find out the return on the portfolio?
(a) Market value of the investment
(b) Beta of the shares
(c) Proportion of Investment
(d) None of the above.

13. Assuming that the portfolio is equally weighted, what is the return on the portfolio?
(a) 12%
(b) 13.5%
(c) 12.75%
(d) 18%

14. A share pays nil dividends and its current market price is 100. The possible selling
prices at the end of the year and the probabilities are:
Price Probability
90 .1
100 .2
110 .4
120 .2
130 .1

What is the expected rate of return at the end of the year?


(a) 8%
(b) 12%
(c) 10%
(d) 11%

15. What is the standard deviation of expected return?


(a) 10.55%
(b) 10.6%

Financial Planning Academy 54


(c) 10.9%
(d) 10.95%

16. A company’s reserves are fast depleting and its sales are declining in recent years. Its’
costs of production is also on the increase. Because of these reasons, the company’s
earnings are declining and dividends are expected to fall by 5% per annum. Current
dividend is Rs. 5 and the discount rate is 15%.
What is the value of the share?
(a) 20
(b) 21.25
(c) 23.75
(d) 19

17. As a financial adviser, under what circumstances would you recommend buying of
the share?
(a) If the company is expected to distribute its large reserves in the near future.
(b) If the company is coming out with a fresh issue to finance its expansion
(c) If the company is taking efforts to cut down its expenses.
(d) None of the above.

18. You buy a share of Hindustan Ltd. for Rs. 25. You expect it to pay dividend of Rs. 5.
You also expect it to bill it at Rs. 32 at the end of the year. What is the dividend yield?
(a) 10%
(b) 15%
(c) 30%
(d) 20%

19. Sudha owns the following portfolio:


Share Weight Beta Expected Return
Asian Ltd. .30 1.20 15%
Bhilwara Ltd. .20 .75 11%
Asian Hotel .50 1.00 12%

What is the expected return on the portfolio?


(a) 8%
(b) 10%
(c) 12.7%
(d) 15%

20. What is the portfolio beta?


(a) .1775
(b) 1.15
(c) 1.01

Financial Planning Academy 55


(d) .9975

21. Alok industries has announced sales forecast of Rs. 15, 50000 for the year 2005. If
the net income margin is 5% and there are 1,00,00 equity shares outstanding, what
would be the projected earning per share
(a) .775
(b) 1.70
(c) 1.77
(d) .70

22. What would be the market price if the P/E of the companies in the same industry is
20?
(a) 15
(b) 35
(c) 15.50
(d) 25.40

23. As per SEBI (Stock Brokers and Sub-Brokers) rules, 1992, a share broker applies for
registration to SEBI through a stock exchange of which he is admitted as a member.
True/False

24. No stockbrokers are allowed to become the member of more than one stock
exchange. True/False

25. XYZ Ltd. currently earns Rs. 5 per share. Its return on equity is 20% and it retains
50% of its earnings. (Both of the figures are expected to be maintained indefinitely).
Stocks of similar risk are priced to return 12%. What is the intrinsic value of the
share?
(a) 130.55
(b) 137.50
(c) 148.25
(d) 155.75

ANSWERS

Financial Planning Academy 56


1. D’ = 8 (1.05) = Rs. 105
R–g .13 - .05

Required Rate of Return is Risk free Rate + Risk Premium

2. C – An increase in Dividend Growth Rate leads to an increase in stock prices

3. A
4. C
5. C
6. B Po = D1 g = r – D1/Po
R-g

7. C
8. C The dividend stream during the first eight years is D1 = 2.00 (1.18)
D2 = 2.36 (1.18) D3 = 2.78 (1.18) D4 = 3.29 (1.18) D5 = 3.88 (1.12) D6
= 4.34 (1.12) D7 = 4.86 (1.12) D8 = 5.45 (1.12)

Present value of this dividend stream is 2.36 (1.15), 2.78 (1.15)2, 3.29 (1.15)3,
3.88 (1.15)4, 4.34 (1.15)5, 4.86 (1.15)6, 5.45 (1.15)7, 6.10 (1.15)8
= Rs. 16.83

P8 = D9 6.10 (1.06) = 71.84


r–g .15 - .06

Present value of this price is = 71.84 = 23.49


(1.15)8
Present value of Dividend + Share = Rs. 16.83 + Rs. 23.49 = Rs. 40.32

9. A Estimate the P/E Ratio = Dividend Payout Ratio


Required Rate of Return – Growth
.5/(.12 - .05) = 7.14

Po = EPs1 x P/E Estimate

5 x 7.14 = Rs. 35.70

10. D First calculate Required rate of return = .08 + 1.5 (.15 - .08) = 18.5%
Po = D1 3 (1.05) = 23.33
R–g .185 - .05

11. A value = Dividend / Rate of Interest

Financial Planning Academy 57


20/.12 = 166.67

12. C

13. B

14. C

15. D

16. C

17. A

18. D

19. C

20. C

21. A

22. C

23. True

24. False

25. B Growth = Return on Equity x Retained Earnings


= .20 x .50
= .10 or 10%

In case of constant growth = E1 x (1 – Plough back Ratio)


R -g

Po = (5 x 1.10) (1 - .50) = 137.50

.12 - .10

Part 8

Stock Exchanges in India follow rolling settlement system.

Financial Planning Academy 58


1. Under the rolling settlement system, any transaction made on a particular day
necessarily results in deliver after a fixed number of days.
(a) True
(b) False

2. Currently stock exchanges in India follow T + 3 settlements:


(a) True
(b) False

3. Switch over from account period settlement system to rolling settlements was
facilitated by which of the following:
(a) Dematerialization
(b) Electronic Fund Transfer
(c) Both a) & b)
(d) None of the above

4. Convertible debentures are valued on the basis of:


(a) Yield to Maturity
(b) Market Value
(c) Cost Basis
(d) Convertible portion as equity investment and non-convertible like debt
investment.

5. Pre-mature withdrawal is not allowed in RBI saving bonds:


(a) True
(b) False

6. Non Resident Indians are not allowed to invest in RBI bonds.


(a) True
(b) False

7. Indexation benefit is considered while computing the capital gain realized on sale of
land or house property held for more than 36 months.
(a) True
(b) False

8. As per section 54 of the Income Tax Act, 1956, there is a long-term capital gain tax
to be paid in case of sale of land or house held for more than 24 months.
(a) True
(b) False

Financial Planning Academy 59


9. The bid ask spread is best described by which of the following statements:
(a) It is broker’s commission
(b) It is dealer’s gross income from a transaction.
(c) It is large for illiquid securities than it is for liquid securities.
(d) All of the above are true.

10. How do treasury bills pay interest to their investors:


(a) Coupon Interest
(b) Possible price appreciation above their discounted price.
(c) T-bills pay no interest.
(d) Difference between issue price and face value.

11. Treasury bills are long-term debt instruments of Central Govt. sold through RBI.
(a) True
(b) False

12. Treasury bills are issued at a discount and redeemed at par.


(a) True
(b) False

13. Commercial papers like Treasury Bills are not issued at a discount are redeemed at
par.
(a) True
(b) False

14. Commercial papers need to be creditable.


(a) True
(b) False

15. With the establishment of National Stock Exchange in 1994, India entered the era of
screen based trading.
(a) True
(b) False

16. Commercial Paper is an instrument of long term secured borrowing issued by non-
banking companies.
(a) True
(b) False

17. Banks & Financial institutions are major issuers of CD, which are transferable.
(a) True
(b) False

Financial Planning Academy 60


18. In the balance sheet of a company, the share capital of investor is shown on the asset
side.
(a) True
(b) False

19. A listed company is required to furnish audited financial results on a quarterly basis
with in a month of the expiry of the period to the stock exchange where it is listed.
(a) True
(b) False

20. The companies’ act has prescribed a standard format for Profit & Loss A/c.
(a) True
(b) False

21. The issue of Govt. securities is regulated by SEBI.


(a) True
(b) False

22. Mutual funds are the largest holders of Govt. securities.


(a) True
(b) False

23. Individuals cannot invest in Govt. securities.


(a) True
(b) False

24. A certain amount of PF money can be invested in approved corporate debentures.


(a) True
(b) False

25. Preferential Allotment refer to date of equity or equity related instrument of a listed
company
(a) True
(b) False

26. Private placement refers to sale of equity or equity related instruments of our
unlisted company or sale of debentures of a listed or unlisted company
(a) True
(b) False

27. Currently for delivery based transactions in equity security transaction tax is
0.30% & the same is to be split between the buyer & the seller.
(a) True
(b) False

Financial Planning Academy 61


28. The basis point is which of the following:
(a) One rupee
(b) One paisa, Re .01
(c) One hundredth of one percentage point, 0.01 of 1 percent
(d) One percent

29. Government bond is best described by which one of the following statements.
(a) It may be resold at any time.
(b) It has no voting privileges
(c) It receives no cash dividends
(d) All of the above

30. Preference shareholders receive priority over common stockholders with respect to
which of the following.
(a) Dividends cannot be paid to common stockholders unless the presence stockholders
receive their dividend.
(b) In the event of bankruptcy and liquidation, the preference shareholders are paid
before the common shareholder.
(c) Preference shareholders get to elect the chairman of the corporation’s Board of
Directors.
(d) All of the above

31. A preference share holder is


(a) A marketable security
(b) It pays fixed dividend
(c) A debt security
(d) Both A & B above

32. Which one of the following statements best describes the corporate bonds?
(a) Bond investors are creditors of the corporation
(b) Majority of bonds make annual coupon interest payments.
(c) Both A and B above are true
(d) None of the above is true.

33. A debenture trust deed is described by which of the following statements.


(a) It is a legal contract describing the right of specific bondholders.
(b) It described the duties of the trustees.
(c) Both A & B are true
(d) None is true

34. The quality rating of a corporation’s bond issue are primarily determined by which
of the following:
(a) The level and trend of the issuer’s financial ratios.
(b) The level and structure of interest rates.

Financial Planning Academy 62


(c) The issuer’s financial condition and indenture contract that governs the issuing
firm.
(d) All of the above.

35. On bond with A grade rating might pay a higher yield to maturity than another A
grade bond issued at a different time by the same corporation due which of the following
reasons?
(a) The bond market is sometimes irrational and evaluates the riskiness of some bond
erroneously.
(b) One bond is a secured one & another unsecured one.
(c) Both a) & b) above
(d) None of the above.

36. Which one of the following bond quality ratings apply to default free bonds?
(a) AAA
(b) AA
(c) Both a) & b) above
(d) None of the above

37. A debt rating essentially reflects the probability of timely payment of interest &
Principal by a borrower. The higher the debt rating, the greater the likelihood that
the borrower will not default in making payment.
(a) True
(b) False

38. NSE is:


(a) Price weighted Index
(b) Value weighted Index
(c) Equal weighted Index
(d) None of the above

39. A value weighted index is best described by which of the following statements:
(a) Equal weights are assigned to each share with index.
(b) The index reflects the aggregate market capitalization of the sample shares on a
certain day in relation to a base year.
(c) The weight assigned to each stock is proportional to its price per share.
(d) None of the above.

40. All stock indexes are most accurately characterized by which of the following
Statements about the degree to which they co-vary together?
(a) They are perfectly positively correlated
(b) They are highly positively correlated

Financial Planning Academy 63


(c) They are uncorrelated.
(d) They are positively correlated.

41. A primary issue of bonds or stocks will increase both sides of the issuing company’s
balance sheet by the same amount.
(a) True
(b) False

42. Which of the following factors intend to increase the growth rate of a corporation:
(a) External Borrowing
(b) Increasing the Retention Rate
(c) Increasing the Rate of Return on Equity
(d) All of the above

43. Imperfect hedges occur when either the quantities sold short and bought long are out
of balance or purchase and short sale price differ.
(a) True
(b) False

44. What characteristics long and short positions have in common?


(a) The potential profits from a long position and the potential losses from a short
position are both infinite if the price of the underlying security rises to infinity.
(b) There is a one to one correspondence between movements in the price of the
underlying security and the investor’s profits.
(c) Both a) & b) are true.
(d) None of the above.

45. Why do share prices usually drop when news about decline in a company’s earning
per shore is reported?
(a) Because a reduction in earnings means that the firm has less money with which to pay
dividends and therefore, the market fears a reduction in the company’s future
dividends.
(b) Because the share market anticipates that a decreased level of earning power might be
an indicator of default and perhaps bankruptcy.
(c) Companies usually do not declare their annual results.
(d) Both a) & b) above.

46. The minimum subscription amount in primary market in case of Govt. securities is:
(a) Rs.1,00,000
(b) Rs.10,000
(c) Rs.1,000

Financial Planning Academy 64


(d) Rs.100

47. Subsidiary General Ledger is:


(a) A facility provided by RBI.
(b) Holding of investment in Govt. securities by RBI.
(c) Necessary for financial institutions, primary dealers and NBFC’s to deal in Govt.
securities.
(d) All of the above.

48. Securities market in India is regulated by:


(a) Dept. of Economic Affairs
(b) Dept. of Company Affairs
(c) Securities Appellate Tribunal
(d) All of the above

49. Companies Act lays down rules & regulations regarding:


(a) Issue, allotment and transfer of securities.
(b) Standard of disclosure in public issue.
(c) Underwriting of issues.
(d) All of the above.

50. The companies in which significant amount of shares are held by the parent which
also exerts influence in the activities is known as:
(a) Wholly owned subsidiaries
(b) Partly owned company
(c) Associated companies
(d) Unlimited company

51. For knowledge companies, the core activity must provide ………of the total income
in the preceding ……years.
(a) Not less than 51%, 3
(b) Not less than 75%, 3
(c) Not less than 75%, 2
(d) Not less than 51%, 2

52. For listing, the company must submit to ……audited balance sheet for the past ……
years.
(a) SEBI, 3
(b) CLB, 2
(c) NSE, 3
(d) None of the above

53. For IPO’s by companies, the paid up equity must be more than … crores and market
capita more than … crores.
(a) 25, 50
(b) 10, 25

Financial Planning Academy 65


(c) 10, 50
(d) 25, 50

54. Which of the following is true as a book building issue?


(a) Demand for IPO is known throughout the issue.
(b) Demand for IPO is known at the end of the issue.
(c) Demand for IPO is known in the beginning of the issue.
(d) None of the above.

55. Which of the following statements is true?


(a) Price discovery is less efficient in a book building issue than in a fixed price IPO.
(b) Price discovery is better in a book building issue than in a fixed price IPO.

56. Which of the following statements are true?


(a) Book building is very common in India.
(b) Book building is less common in developed than emerging markets.
(c) Book building is more common in developed than emerging markets.
(d) None of the above.

57. For the Financial year 2005-2006, what is true about the dividend on equity
schemes?
(a) The recipient is required to pay tax at normal rates.
(b) The recipient is not required to pay tax on dividend received by him.
(c) The distributing company is required to pay distribution tax.
(d) None of the above.

58. For the financial year 2005-06, what is true above dividends on debt schemes?
(a) The distributing company is required to pay dividend distribution tax.
(b) The recipient has to pay tax on dividend received by him.
(c) The recipient is required to pay tax at marginal rate.
(d) None of the above.

59. Real estate market in India is:


(a) Highly disorganized
(b) Disorganized
(c) Is free from Govt. controls
(d) Offers homogeneous products

60. What is the Indian Housing Society?


(a) Lack of clear title in most cases.
(b) High stamp duty rates.
(c) Obsolete tenancy and rent control laws.

Financial Planning Academy 66


(d) All of the above.

61. Ashish proposes to purchase a property for giving it on rent. He expects to receive
Rs.55,000 in net receipts each year for six years and to sell the property for
Rs.8,50,000 at the end of six year period. If the expected return is 15%, what should
be the value of property?
(a) 8,00,000
(b) 5,75,625
(c) 7,57,600
(d) None of the above

62. For an asking price of Rs.8,00,000 with an estimated net income of Rs.75,000 p.a.,
at a market yield of 11%. Calculate the value of property on capitalization approach?
(a) 7,00,000
(b) 6,81,818
(c) 7,19,555
(d) 8,81,818

63. A rupee one lakh Treasury bill is selling at Rs. 95000 today. It will mature in 100
days. Calculate the annual yield.
(a) 18.50%
(b) 24.15%
(c) 18.25%
(d) None of the above {(100000/95000)-1 *365/100 }

64. Ram has bought the share of ABC Ltd. For Rs.80,000. They grow to Rs.90,000 in
182 days. What is the return?
(a) 12%
(b) 15%
(c) 12.5%
(d) 12.8%

65. What is the annualized return?


(a) 25%
(b) 25.06%
(c) 28%
(d) 19%

Solution
21. B (It is regulated by RBI)

Financial Planning Academy 67


22. B (Banks are the largest holders of Govt. Securities. About 1/3rd of
and partly for investment purposes).

27. B (The STT is 0.20% and it is to split equally between the buyer and
the seller).

62. B Net Income = 75000 = 681,818


Yield 11%

Part 9

1. Which of the following factors will increase in the duration of a corporate bond?
(i) An increase in the number of years to maturity

Financial Planning Academy 68


(ii) A decrease in coupon rate
(iii) Change from Annual to Semi-annual coupon payment
(iv) Change from Annual coupon to zero coupon

(a) (i), (ii) and (iii) only


(b) (i), (ii) and (iv) only
(c) (i), (iii) and (iv) only
(d) (ii), (iii) and (iv) only

2. The following are long-term credit ratings given by the CRISIL. Which is the lowest
investment grade category?
(a) AAA
(b) A
(c) BBB
(d) B

3.Which of the following products is designed to provide both growth and income?
(a) Fixed Premium Annuity
(b) Aggressive growth mutual fund
(c) Fixed Deposit of Banks
(d) Convertible Bond

4.Which one of the following best describes as ‘debenture’?


(a) A long term corporate Promissory Note
(b) An investment in debt of another corporate party
(c) A corporate debt obligation that allows the holder to repurchase the security at
specified dates before maturity
(d) Unsecured corporate debts

5. Rs. 1000 bond originally issued at loan maturing in exactly 10 years bears a coupon
rate of 8% compounded annually. It is currently selling at Rs. 1147.20. The indenture
agreement provides that the bond may be called after five years at Rs. 1050/-. Which of
the following statements are true?
(a) The yield to maturity is 6 %
(b) The yield to call is 5.45%
(c) The bond is currently selling at a premium indicating that the market rates have fallen
since the issue date
(d) All of the above are true.

6.Which of the following are non-diversifiable risks?


(i) Business risk
(ii) Management risk
(iii) Company or industry risk

Financial Planning Academy 69


(iv) Market Risk
(v) Interest Rate Risk
(vi) Purchasing Power Risk

(a) (iv), (v) and (vi) only


(b) (i), (ii) and (iii) only
(c) (ii), (iii) and (v) only
(d) None of the above.

7. To immunize a bond portfolio over a specific investment horizon, an investor would do


which of the following?
(a) Match the maturity of each bond to the investment horizon
(b) Match the duration of each bond to the investment horizon
(c) A and B above
(d) Match the average weighted duration of the bond portfolio to the investment horizon.

8. Assuming that the current market yield for similar risk bonds is 8%, determine the
discounted present value of a Rs. 1000 bond with a 7.5% coupon rate which pays interest
semi annually and matures in 17.5 years
(a) Rs. 504.68
(b) Rs. 539.78
(c) Rs. 953.34
(d) Rs. 1653.26

9.Bond A has 6% annual coupon and due in 2 years. Its value today in the market is Rs.
900. Its yield to B, maturity is:
(a) 9.90%
(b) 10.40%
(c) 10.90%
(d) 11.90%

10. A 10 year annual annuity has a yield of 9 percent. What is its duration?

11. The following information is available for a bond:


Face value = 100
Coupon Rate = 9 percent payable annually
Years to Maturity = 5
Yield to Maturity = 7.75%
Current Market Price = 105
What is the duration of the bond?

12.Which of the following best describes the investment characteristics of a high quality,
long term Govt. bond?
(a) High inflation risk; low default risk
(b) Low inflation risk, high market risk

Financial Planning Academy 70


(c) Low inflation risk, low default risk
(d) High inflation risk, high market risk

13.Duration of a zero coupon bond is less than its terms to maturity


True/False

14. A longer maturity period bond is less sensitive to change in interest rates.
True/False

15. A large coupon-paying bond is less sensitive to change in interest rates.


True/False

16 Duration for a coupon-paying bond is always less than its term to maturity.
True/False

17.The duration of a coupon-paying bond varies inversely with its yield to maturity.
True/False

18. When a bond is selling at a discount, its YTM exceeds the coupon rate.
True/False

19. If market interest rate falls, a coupon-paying bond will not experience an increase in
duration.
True/False

20. A bond’s duration measures which one of the following:


(a) Time structure of bond’s cash flows
(b) The bond’s interest rate risk
(c) Both A & B above
(d) The default risk of the bond issue

21.Bonds with higher coupon, other things being the same


(a) Have more interest rate risk than bonds with smaller coupons
(b) Have less interest rate risk than bonds with smaller coupons
(c) Have lower duration than smaller coupon bonds
(d) Have higher duration than smaller coupon bonds

22.For a given coupon rate, a bond’s duration generally increases with maturity.
True/False

23.Bond prices and interest rates are directly related.

Financial Planning Academy 71


True/False

24. The duration of a bond, in effect, represents the length of time that elapses, before, the
“average” rupees of present value from the bond is received.
True/False

25.The duration of a zero coupon bond is less than its maturity.


True/False

Answers

10. 4.79 years

Financial Planning Academy 72


1 + Yield - Number of Payments
Yield 1 + Yield (No. of Payments)

1.09 - 10 = 4.79 years


.09 (1.09)10 – 1

11 4.26 years

Year Cash Flow Present Value Proportion of the bond value


at 7.75% X times
1 9 8.35 8.35
2 9 7.75 15.50
3 9 7.19 21.57
4 9 6.67 26.68
5 109 75.04 375.20
105.00 447.30

447.30 = 4.26 years


105

Part 10

Read the data below and the answer the following questions (1-7):

Financial Planning Academy 73


(Time to Mature)
Share Current Price Exercise Price Call Premium Put Premium
2months 3months 2months 3months
X 35 37 2 3 .50 1.50
Y 30 34 1 1.25 5.50 6.00
Z 25 30 3 3.30 2.00 2.20

Each contract is equal to 100 shares.


1. 1. If you purchase a 3-month call contract on X, What profit or loss do u make if the
price of X at maturity is Rs.42?
(a) +300
(b) +200
(c) +400
(d) -200

2. If Y’s market price at 3 months is 37, determine the profit or loss of 3 months by put
option.
(a) +900
(b) -600
(c) +600
(d) -900

3. If Y’s market price at 2months is 37, determine the profit or loss to put writer:
(a) -550
(b) +550
(c) +850
(d) -300

4. If you have sold four 3 months put option on Z and Z’s price at maturity is 28, what is
your profit or loss?
(a) -800
(b) +880
(c) + 80
(d) -880

5. If you had bought four 2months by option on X and X’s price at maturity is 45, what
is your profit or losses?
(a) -2400
(b) -3200
(c) +2400
(d) +3200

6. Your client wrote five, 2months put option on Y and Y’s price at the spot is 37, what
is his profit or loss?
(a) +1500
(b) -1500

Financial Planning Academy 74


(c) +2750
(d) -2750

7. If you had purchased five 3months call options each on X, Y and Z and the stock
price at present is 35, which of the following options are in the money?
(a) X
(b) Y
(c) X and Y
(d) Z and Y

8. A call option at a strike of Rs.176 is selling at a premium of Rs.18. At what price will
it break even?
(a) Rs.196
(b) Rs.204
(c) Rs.187
(d) Rs.194

9. A stock is currently selling at Rs.50. The call option to buy the stock at Rs.49 costs
Rs.4. What is time value of the option?
(a) Rs.4
(b) Rs.5
(c) Rs.3
(d) Rs.2

10. Spot value of Nifty is 1140. An investor buys one month Nifty 1157 call option for a
premium of Rs.7.The option is:
(a) In the money
(b) At the money
(c) Out of the money
(d) None of the above

11. On 15th January, Nitish bought one January Futures Contract which cost him
Rs.2,69,000. On this, he had to pay an initial margin of Rs.21,520 to his broker. Each
Nifty future contract is for delivery of 200 Nifties. On 25th January, the index closed
at 1280. How much profit or loss did he make?
(a) +13000
(b) -12500
(c) -13000
(d) +12500

12. Nifty spot stands at1260 and the cost of financing is 12% per year. What is the fair
value of one month Nifty futures contract?
(a) 1262
(b) 1272

Financial Planning Academy 75


(c) 1268
(d) 1275

13. What is the fair value of one-month future if the spot value of Nifty is 1150?
The money can be invested at 14% p.a. and Nifty gives a dividend yield of 4% p.a.
(a) 1162
(b) 1159
(c) 1180
(d) 1170

14. A speculator hopes that HCL is going to rise sharply. He has a long position on HCL
in the market of Rs.1 crore. The beta of the stock is 1.2. Which of the following
positions on index futures give him a complete hedge.
(a) Long Nifty Rs.1crore
(b) Short Nifty Rs.1crore
(c) Long Nifty Rs.1.2 crore
(d) Short Nifty Rs.1.2 crore

15. You are a speculator. You feel that the market will be volatile in the next 3 months.
However, you have no idea if it will move upwards or downwards. To take
advantage of the volatility, you would buy:
(a) Three months call
(b) Three months put
(c) A three months call and a three months put with the same strike.
(d) A three months call and sell a three month put with the same strike.

16. A portfolio is composed of Rs.1000 invested in a stock with a beta of 1.1 and Rs.1000
in a stock with a beta of 0.8. What is the portfolio beta?
(a) .085
(b) .90
(c) .95
(d) 1.0

17. Jennifer is optimistic of the long-term growth of her HCL stock. The stock Currently
priced at Rs.58, has made a sharp advance in the last week and she wants to lock in a
minimum price incase the shares drop. What might Jennifer do?
(a) Buy Rs.55 call options
(b) Sell Rs.55 call options
(c) Buy Rs.55 put options
(d) Sell Rs.55put options

18. A client with a large, well diversified common stock portfolio express concern about
a possible market decline. However, he/she does not want to incur the cost of selling a
portfolio of holding nor the risk of mistiming the market. A possible strategy for
him/her would be:

Financial Planning Academy 76


(a) Buy an index option
(b) Sell an index option
(c) Buy an index put option
(d) Sell an index put option

19. A call option with a strike price of Rs.110 has been taken at a premium for Rs.3. The
market price of the underlying stock is Rs.108 today. The intrinsic value of the call is:
(a) 0
(b) 1
(c) 2
(d) 3

20. Which of the following strategies can cause the investor to experience the greatest
loss?
(a) Selling a naked put option
(b) Selling a naked call option
(c) Writing a covered call
(d) Buying a call option

21. Answer the following questions with regard to index options trading on the stock
exchanges in India.

( I )The options are settled


(a) By physical delivery
(b) In cash
(c) Gains/losses are settled in cash and the contract value by physical delivery.
(d) None of the above

( II ) The option contracts have a trading cycle of:


(a) 1month
(b) 6months
(c) 2months
(d) 3months

22. Protective option (call or put) means:


(a) Holding only call or put options
(b) Having the securities and no options
(c) Holding securities and writing the call
(d) Holding securities and writing the put

23. Naked position in derivations means:


(a) No derivative instrument in the portfolio
(b) Having the securities and equivalent derivative instruments
(c) Holding securities and buying the call

Financial Planning Academy 77


(d) Only call or put or future open position

24. Your financial adviser tells you that trading in derivatives is less risky than selling
short. What would be your strategy to benefit from your expectation?
(a) Write a call
(b) Write a put
(c) Long call
(d) Long put
(e) Either A or D

25. For hedging large portfolios in the event of market decline, which of the following
would be suitable?
(a) Derivation on stocks
(b) Derivatives on index
(c) Exchange Traded Funds
(d) Both A & B above

26. Intrinsic value of call option is:


(a) Max (0, X-S)
(b) Min (0, X-S)
(c) Max (0, S-X)
(d) Min (0, S-X)

27. Intrinsic value of put option is:


(a) Max (0, X-S)
(b) Max (0, S-X)
(c) Min (0, X-S)
(d) Min (0, S-X)

28. Benefits of trading in future contract are:


(a) Leveraging exposure
(b) Ease of exposure
(c) Lower transaction costs
(d) All of the above

29. Which of the following conditional orders can be placed with the brokers/trading
members as per their requirements?
(a) Orders based on time conditions
(b) Orders based on price conditions
(c) Both A & B above
(d) None of the above

30. “Marked to Market” means that the profits and losses on future contract are settled on
a periodic basis.
(a) True
(b) False

Financial Planning Academy 78


31. Mutual funds are not allowed to trade in derivatives.
(a) True
(b) False

32. Dividends distributed by mutual funds are not tax exempt in the hands of the
recipient.
(a) True
(b) False

33. As per the SEBI regulations, if a scheme is in existence for over a year, compound
annual yield is the accepted method of calculating return and returns should be shown
for the last 1,3 and 5 years of the scheme or since its inception.
(a) True
(b) False

34. The AMC of a Mutual Fund is appointed by:


(a) Trustee
(b) Sponsor
(c) Custodian
(d) Either Sponsor or Trustee

35. The investment management fees charged by the AMC for managing the assets of the
mutual fund is regulated by SEBI.
(a) True
(b) False

36. A mutual fund under all its schemes taken together will not own more than 5% of any
company’s paid up capital carrying voting rights.
(a) True
(b) False

37. A Mutual Fund can borrow by offering the assets of the scheme as collateral for
borrowing, to meet the liquidity needs for the purpose of re-purchase, redemption of
units or payment of int. or dividend to the unit holders.
(a) True
(b) False

38. A mutual fund shall not invest more than 10% of its NAV in any unlisted security of
an associate or group company of the sponsor.
(a) True
(b) False

Financial Planning Academy 79


39. Which of the following legislations are applicable to Mutual Funds in India?
(a) Companies Act, 1956
(b) SEBI Act, 1992
(c) Indian Trust Act, 1882
(d) All of the above

40. The custodian is appointed by:


(a) Asset Management Company
(b) SEBI
(c) Board of Trustees
(d) Sponsors

41. What is the maximum contribution of sponsor to net worth of Asset Management
Company?
(a) Maximum 40%
(b) Minimum 40%
(c) Maximum 75%
(d) Minimum 25%

42. Mutual Funds are allowed to invest in overseas securities, subject to approval of
Board, without any investment cap or ceiling.
(a) True
(b) False

43.A mutual fund’s balance sheet is as follows:


Liabilities Rs. Crores Assets Rs. Crores

Financial Planning Academy 80


Unit Capital Equity Shares 900
(50,00,00,000 units at Rs.10 each)500 Debentures 350
Reserves & Surplus 850 Money Market Instruments 145
Current Liabilities 150 Other Current Assets 105__
Total 1500 1500

What is the net asset value of a unit?

44. Mutual Funds can borrow money for a period:


(a) Not exceeding one year
(b) Not exceeding 3 months
(c) Not exceeding 6 months
(d) For any period

45. A mutual fund scheme can invest in another scheme of same Asset Management
Company.
(a) True
(b) False

Solution

12. 1260 x (1.12) 1/12 = Rs.1272

Financial Planning Academy 81


13. 1150 x (1 +.14 - .04)1/12 = Rs.1159

35. A (Fees can be on the first Rs.100 crores of the weekly average of
the net assets 1.25%, on the balance of the assets = 1%)

36 B (The limit is 10%)

38 B 38 (The limit is 25%)

43 Net Assets = Investments + Current Assets – Current Liabilities


= 1500 – 150 = 1350 cores
NAV = 1350_
50
= Rs.27

Financial Planning Academy 82


Part 11

Q1. Praj industries has declared dividend of Rs 5 per share for the current year. The
dividend is expected to be same for the following year, increasing to Rs 6 the year after.
From there on , the dividend is expected to grow at 5% per year indefinitely. Stock with
similar risk are currently priced to provide a 10% expected return. What is the intrinsic
value of Praj Industries.

Q2. Mukesh Ltd Currently earns Rs 5 per share its return on equity is 20% and it retains
60 % of its earning ( both figures are expected to be maintained indefinitely ). Stock of
similar risk are priced to return 15%

Q3. A non dividend paying stock has a current price of Rs. 20. What will be the future
price if the risk free rate is 10% and the maturity of the future contract is 1 month ?

Q4. Suppose a stock index has a current value of 4000. If The risk- free rate is 9%
and the expected yield on the index is 3%, what should be the price of a 6 months
maturity future contract?

5. NAV of one unit of a mutual fund is Rs 11. The Exit load is 4%. The Sale price to the
investor would be
(a) Rs 11
(b) Rs 11.44
(c) Rs 10.56
(d) Rs 11.50

6. As per Markowitz, a 50- asset portfolio has ____ unique covariance terms.
(a) 2499
(b) 1449
(c) 1225
(d) None of these
( Covariance terms = n(n-1)/ 2)

7. Bond price volatility is directly related to the bond’s coupon.


Ans. False ( It is inversely to yield )

8. For any given maturity, bond price movements that result from an equal absolute
decrease or increase in the yield – to- maturity are symmetrical.
Ans. False

9. Duration for a coupon –paying bond is always less than its term to maturity
Ans. True

10. When a bonds YTM equals its coupon rate , the bonds price is less than per value.
Ans. False

Financial Planning Academy 83


11. Ramesh Khanna owns a portfolio whose market model is expressed as :
rp = 2% + 0.90ri + epi
Where ri= Return on the market index
epi = error term

If the expected return on the market index is 12 %, what is the expected return on
Ramesh Portfolio?

Ans:- With 12% expected return on the market index, the market mode would imply that
the expected return on Ramesh’s Portfolio would be :

rp = 2% + .90*12

rp= 12.3 %

12. Which of the following is true :


(a) Agriculture income is exempt from income tax
(b) Agriculture income is exempt from wealth tax
(c) Real estate investment is illiquid
(d) All of above

13. Risk free rate of return 6%


Expected rate of return on market portfolio 18%
Beta of Security A 0.7
Beta of Security B 1.4
Answer the following question based on the above mention information

I. The Expected rate of return of security A will be


(a) 14
(b) 13.6
(c) 16
(d) 14.4

II. The Expected rate of return of security B will be


(a) 21
(b) 19.85
(c) 22.8
(d) 20

III. If a security has expected return of 20% , its beta will be


(a) 1.167
(b) .9
(c) 1
(d) 1.5

Financial Planning Academy 84


IV. If the risk free return is 8% and the expected return is 20% , beta will be
(a) 1
(b) 1.85
(c) .20
(d) 1.20

14. The risk – free rate is 8% and the expected return on the market portfolio is 15%. The
beta of stock A is 1.2 . Investor believe that the stock will provide an expected return of
18% . What is the Alpha as per Security Market Line?

Ans :- Alpha = Actual expected return – Fair return

Fair return of Stock A = Risk free + beta of stock A ( Market return – Risk free)
= 8 + 1.2 ( 15- 8 )
= 16.4 %

Alpha of Stock A = 18-16.4 = 1.6%

15. Your friend offer to sell equity shares of a company that paid dividend Rs. 5. You
expect the dividend to grow at the rate of 7% p.a for the next 5 year. If your holding
period is 5 years and opportunity cost is 12 %, find out

I. Present value of the expected dividend in the 5 years


(a) 20
(b) 19
(c) 22
(d) 21.85

II. At the end of the holding period, the share price is expected to be Rs.35. Find out the
Present value of the expected price
( Growth rate is considered as discounting )

III. If the current market price of the stock is 37, what would you decide
Buy and Hold the share
Advise your friend to sell the share in the market.

16. Which of the following advantage is not available to a person investing in mutual
funds
(a) Control over actual investment
(b) Selection flexibility to invest or withdraw funds
(c) Transparency
(d) Convenient administration
(e) All are available

Financial Planning Academy 85


17. You are given the following information about stock X & Stock Y
(Rx – Rm)= 2% + 1.5( Rm – Rf)
(Ry – Rm)= 3% + 1.0( Rm- Rf )

I. Rate of Risk free security is 7%. If the market portfolio is expected to generate 14% for
the year, what is the Expected return on Stock X?
(a) 12%
(b) 16.5%
(c) 15%
(d) 17.5%

II. What is the Expected return on Stock Y


(a) 19%
(b) 18%
(c) 16.5%
(d) 17.5%

18.Following Question are based on the mutual funds borrowing.

I. Mutual Funds can borrow money for a period of


(a) Not exceeding one year
(b) Not over three months
(c) For any period
(d) Not exceeding six months

II. Which of the following statement is not correct


(a) SEBI restricts the quantum of borrowings by mutual funds.
(b) Corpus can be increased through borrowing
(c) Temporary liquidity needs can be met through borrowings

19. If you are planning an investment in a remote, rural area where you have a hunch that
the national high way would come up in the vicinity and the place would be a township
preferred by BPO entrepreneurs. What is this category of real estate investment?
(a) Passive secure investment
(b) Development investment
(c) Speculative investment
(d) Both a & b

20. In the Primary market, the Fixed income securities are issued through
(a) Public issue
(b) Private placement
(c) Tender / Auction
(d) Any of the above

Financial Planning Academy 86


21. Suppose you have invested in 2000 shares of an Wipro company and you are afraid
that the price would decline. What would be your strategy to protect the value of your
portfolio ?
(a) Sell a put option
(b) Buy a put option
(c) Buy Futures

22. Can a Portfolio be hedged with a call option ? If you can, what would you do.?
(a) Buy call option
(b) Hedging requires a combination call and put option
(c) Sell call option
(d) Hedging through call option not possible.

23. For an investor taxation has always been an important consideration .Based on these
answer the following question .

I. Cost of acquisition is
(a) Value spent for acquiring the investment plus depreciation
(b) Value spent on acquiring the investment minus the capital expenses incurred for the
acquisition.
(c) Value spent on acquiring the investment minus the capital expenses incurred for
the acquisition.

II.. Which of the following statements is correct


Cost of acquisition of bonus share is nil.
Cost of acquisition of bonus share is equal to the market value of share on the date of
allotment.
Cost incurred on transfer of shares is excluded in full value consideration computation.

(a) Statement A and B is correct.


(b) Statement A is correct , and statement B is wrong.
(c) None of them is correct.

Financial Planning Academy 87


FINANCIAL PLANNING ACADEMY

Mock Test - I

Question 1: All of the following are major steps in the asset management process except

A: Analyzing information.
B: Developing an investment policy statement.
C: Establishing goals.
D: Monitoring performance.

Question 2 :
Anita D'sa financial situation is as follows:

Cash/cash equivalents 5,000


Short-term debts 3,000
Long-term debts 121,000
Taxes 8,000
Invested assets 45,000
Use assets 192,000
What is her net worth?

A: 110,000
B: 118,000
C: 124,000
D: 242,000

Question 3 : Which one of the following statements best describes the income statement
and its major components?

A: The income statement indicates, as of a definite date, the client's receipts and
disbursements. Its components include salaries, wages, and expenses.
B: The income statement indicates, for a certain period of time, an individual's cash
inflows and outflows. Its components include salaries and wages, expenses, and surplus
or deficit.
C: The income statement indicates, as of a definite date, what an individual owns and
owes. Its components include cash equivalents, use and personal assets, liabilities, and
net worth.
D: The income statement indicates, for a certain period of time, the growth of a client's
assets and liabilities. Its components include cash equivalents, use and personal assets,
liabilities, and net worth.

Financial Planning Academy 88


Question 4 : As part of her preparation to make suitable recommendations to a pair of
married clients, investment professional Sonya Khan determines that the couple cannot
accumulate the amount of capital they wish by the time they retire. The problem stems
from their aversion to risky investments and the limited amounts they expect to make
available as periodic investments. Before making a recommendation, Sonya should do
which one of the following?

A: Make it clear that they will have to accept higher risk in their portfolio.
B: Advise that they increase their periodic investments to reach their stated accumulation
goal.
C: Explain the problem and seek clarification on how they wish to adjust their
accumulation goal, risk posture, or periodic investments.
D: Counsel the couple that they must reduce their accumulation goal.

Question 5 : The overall purpose of an investment policy is to

A: Determine the types of investments the client prefers and the types the client wishes to
avoid.
B: Establish and prioritize the client's investment goals.
C: Indicate, in writing, the client's time frame, risk tolerance level, and particular
circumstances that need to be considered in the investment recommendations.
D: Delineate investment guidelines that match the realities of the client's objectives,
investments, and investment markets.

Question 6 : The investment decision should be made by

A: The investment professional.


B: The client.
C: The investment professional and the client together.
D: All of the above.

Question 7 : Sara Johson is highly methodical and analytical with respect to her
investment program. Which one of the following best describes Sara's investor
personality?

A: Individualist
B: Adventurer
C: Celebrity
D: Guardian

Financial Planning Academy 89


Question 8 : Of the following, which one describes a key attribute of an investment
policy?

A: Definitive- specific securities identified


B: Clearly defined- could be managed by a competent stranger
C: Flexible- most decisions on securities are left to the investment professional's
discretion, based on the investment policy
D: All of the above.

Question 9 : In the early stages of investment policy development, the client must

A: Defer to the greater experience of the investment professional.


B: Take a leading role.
C: Share responsibility equally with the investment professional.
D: Merely provide the necessary documentation about his or her financial situation.

Question 10 : In periods of market volatility, how can the investment professional add
value to client relationships?

A: Separate random noise from true change


B: Motivate the client to move out of the market in its early stages of decline
C: Establish a new investment policy for the client
D: All of the above.

Question 11: Tarun Jha’s plans to invest all of his money in the stock market, even
though valuations are at a very high level, because the market has been in a strong bull
market for the past three years. Terry is displaying

A: Representativeness.
B: Fear of regret.
C: Fiduciary fear.
D: All of the above.

Question 12 : Chetan Kapadia read a newspaper article that reported that ABC Growth
Fund was up 20% last six months. Based only on that strong performance, Chetan
decides to buy that fund. What possible investor mistake is Chetan making?

A: Overconfidence
B: Mental accounting
C: Rationalization
D: Recency

Financial Planning Academy 90


Question 13 : SRK’s favorite stock dropped on news of additional debt issued by the
company. To which one of the following unsystematic risks is SRK's stock most likely to
be subject?

A: Interest rate risk


B: Default risk
C: Financial risk
D: Reinvestment rate risk

Question 14 : A measure of the variability of a security's return is

A: Standard deviation.
B: Beta.
C: The Sharpe index.
D: The Jensen index.

Question 15 : The risk-free rate is 3%, the market rate of return is 11%, the standard
deviation of XYZ stock is 20, and the beta of XYZ stock is 1.20. Using the capital asset
pricing model in conjunction with this information, what is the expected return of XYZ
stock?

A: 9.6%
B: 12.2%
C: 12.6%
D: 13.2%

Question 16 :
Assume each of the asset classes below has the following correlation to government
bonds:
Treasury bills: .32
Corporate bonds: .94
Large stocks: .17
Mid-cap stocks: .13
Which one of the following correctly states the impact of diversification on a portfolio of
long-term government bonds?

A: Treasury bills provide more diversification than mid-cap stocks.


B: Mid-cap stocks provides more diversification than large stocks.
C: Corporate bonds provide more diversification than large stocks.
D: Corporate bonds provide more diversification than Treasury bills.

Financial Planning Academy 91


Question 17 : If you wanted to determine if a portfolio did better or worse than could be
expected based on the risk incurred in managing that portfolio, you would use the

A: Sharpe index.
B: Treynor index.
C: Jensen index.
D: All of the above.

Question 18 : Portfolio A had a Sharpe index of 1.40, while Portfolio B had a Sharpe
index of .70. Based on this information, which one of the following statements is correct?

A: Portfolio A had better performance than Portfolio B.


B: Portfolio A had worse performance than Portfolio B.
C: Portfolio A had twice the performance of Portfolio B.
D: Portfolio A had better performance than Portfolio B on a risk-adjusted basis.

Question 19 : Which one of the following explains the variance of portfolio total
returns to the greatest degree?

A: Security selection
B: Market timing
C: Asset allocation policy
D: All of the above.

Question 20 : Investment professional J.ASTHANA has just received the latest long-
term total return data for different asset classes. He sees that common stocks returned
10% compounded with a standard deviation of 14.0; T-bills had a 4% return with a
standard deviation of 3.0. What is the expected return of a portfolio of 30% stocks and
70% T-bills?

A: 8.2%
B: 7.0%
C: 5.8%
D: None of the above.

Question 21 : Strategic asset allocation attempts to

A: Maximize returns on short- to medium-term portfolios.


B: Create efficient portfolios that provide the best balance risk and return over the long
term.
C: Use market timing techniques to maximize performance.
D: Use hedging techniques to minimize downside risks.

Financial Planning Academy 92


Question 22 : To calculate a current yield for a bond, the annual interest payment is
divided by which one of the following?

A: Face value
B: Purchase price
C: Market price
D: All of the above.

Question 23 : Company XYZ has earnings this year of 1.50 per share, expects earnings
next year to increase by 12%, and generally trades at a 20% premium to the S&P 500
index. Currently, the P/E of the S&P 500 index is 20. What is the expected value of XYZ
next year?

A: 33.60
B: 36.00
C: 40.32
D: None of the above.

Question 24 : Which one of the following correctly lists characteristics of the late stages
of the expansionary period in the business cycle?

A: High interest rates, high factory utilization rates, increased wage demands
B: Declining interest rates, high unemployment, expansive Fed monetary policy
C: Tax cuts, wage cuts, high inventories
D: Low inventories, low factory utilization, low interest rates

Question 25 : Which one of the following best describes the requirements of an


investment strategy?

A: Goal, opportunity, cooperation


B: Goal, method, competencies, resources
C: Commitment, method, opportunity
D: Ability, method, favorable environment

Financial Planning Academy 93


Question 26 :
Mahesh Bhatt has adopted a value averaging strategy of increasing he account's value by
5,000 each month. How many shares (rounded to the nearest whole share) should he
purchase in the second month, given the information provided below?
Amount Shares Shares Total Total
Month Price/Share Invested Bought Owned Invested Value
1 5.00 5,000 1,000 1,000 5,000 5,000
2 7.00

A: 400
B: 429
C: 714
D: All of the above.

Question 27 : Which one of the following indicators would a contrarian investor


interpret as a bullish indicator?

A: Large cash positions in mutual funds


B: Decreased short selling
C: Low put-call ratio
D: All of the above.

Question 28 : When using a ladder strategy, an investor would

A: Follow a rupee- cost averaging approach.


B: Buy short-term bonds and long-term bonds.
C: Stagger bond maturities.
D: All of the above.

Question 29 : An enemy of effective strategy is

A: Unrealistic expectations.
B: An emotional, undisciplined client.
C: Inadequate time horizons.
D: All of the above.

Financial Planning Academy 94


Question 30 : Growth stocks typically have which of the following characteristics?

A: High profit margins


B: High betas
C: Small dividends
D: All of the above

Question 31 : The amount remaining after making allowed adjustments to total income
is the definition of which one of the following?

A: Adjustments to income
B: Adjusted gross income
C: Taxable income
D: Net income

Question 32 : Which one of the following is a tax preference item or adjustment for
purposes of the alternative minimum tax?

A: All medical expenses


B: Tax-exempt interest on all municipal bonds
C: Accelerated depreciation of business assets
D: Interest paid on a margin account

Question 33 : Which one of the following best explains why a client would select a
single life annuity distribution option from a retirement plan?

A: To ensure that at least the full value of the annuity based on the client's own life
expectancy will be paid out regardless of when the client dies
B: To maximize current income when the client is in a high tax bracket and has a spouse
who is wealthy in his or her own right
C: To maximize the periodic payment if the client had no need or desire to have any
portion of the plan benefits left after death
D: To ensure that the number of payments made will at least equal his or her life
expectancy based on single life annuity tables

Financial Planning Academy 95


Question 34 : Life insurance can be used as an effective tool for all of the following
except

A: Transferring a business upon the death of the owner


B: A source of retirement funds.
C: Providing operating liquidity to a business owner.
D: Paying the outstanding liabilities of an estate.

Question 35 : Which one of the following generally is true with respect to the life
insurance needs of the typical family?

A: The need for life insurance remains steady until age 65.
B: Life insurance generally allows families to reduce their income taxation exposures.
C: The need for insurance to cover both final expenses and lost income decreases at age
70.
D: Total life insurance requirements may decrease as the individual approaches
retirement.

Question 36 : Assume a client has 30,000 in a money market fund, 2,000 in a bank
account, a mortgage loan of 110,000 (not to be paid off), an auto loan of 20,000, and
credit card balances of 5,000. Also assume that if he died, postmortem expenses would be
15,000. What would be the cash requirements for this client?

A: 3,000
B: 8,000
C: 10,000
D: 118,000

Question 37 : Mrs. Barma purchased a life insurance policy that will pay 100,000 to her
beneficiary should she die at any time during the next year. The policy guarantees the
right to renew this policy every year for the next five years. Mrs. Barma has purchased a

A: Variable life policy.


B: Term life insurance policy.
C: Whole life policy.
D: Universal life policy.

Question 38 : With universal life which of the following is fixed?

A: Face value
B: Cash value
C: Premiums
D: None of the above

Financial Planning Academy 96


Question 39 : Which one of the following is a source of ethical conflict in the securities
industry?

A: Competition within the industry


B: Monthly production expectations
C: Performance advertisements in the media
D: Desire for the client to "beat the market"

Question 40:
A newly licensed registered representative in your office was interviewed by a
prospective client. The client indicated she wanted to invest 50,000 in a safe investment.
The representative stated, "You should try Treasury bonds because they have no risk." A
few days later the prospect became a client and bought 50, 000 of Treasury bonds. Which
ethical standard of conduct did the representative violate?

A: Objectivity
B: Fairness
C: Duty to disclose
D: Professionalism

Question 41: A prospective client calls an investment advisor requesting information


about a corporation bond fund that your firm is offering for sale. The advisor quickly
sends a prospectus for the clients review and makes an appointment for a follow-up
telephone call the next week to finalize purchase arrangements. Which one of the
following ethical duty violations has occurred?

A: Duty to disclose
B: Duty to diagnose
C: Duty to consult
D: Duty to keep current

Financial Planning Academy 97


Question 42:
Danny is married and has two children: Ella, 14, and Jessica, 12. He is concerned about
providing enough life insurance to cover his children's education, estimated to be 150,000
plus 220,000 for living expenses until the children reach age 18. In addition, he wants his
wife to have 20,000 in an emergency fund upon his death. He has a 100,000 mortgage, a
10,000 auto loan, a 5,000 balance on his credit cards, and funeral and other death-related
expenses are estimated at 15,000, all of which he wants to pay off at the time of his death.
His liquidity consists of 12,000 in a money market fund.
Based on this information, how much life insurance does Danny need to meet his
objectives?

A: 370,000
B: 485,000
C: 508,000
D: 520,000

Question 43 : The yield to maturity on a bond is

A: Is always less than its current yield


B: Increases as the bond's premium increases
C: Is the annual interest payments divided by the current bond price
D: Factors in any premium or discount of the bond with the annual interest earned on the
bond over the life of the bond

Question 44:
If your client owns the following portfolio, what is their total portfolio expected return?
Assets Amount of Expected
Investment Return

Cash Equivalents 20,000 4.5%

Long-term bonds 60,000 7.5%

Common stocks 110,000 15.5%

Foreign stocks 40,000 13.0%

A: 8.9%
B: 11.1%
C: 12.0%
D: 13.9%

Financial Planning Academy 98


Solution :- ( Mock Test – I)

Answer 1: The correct answer is B.

Developing an investment policy statement is a component of the asset management


process but not one of the major steps.

Answer 2: The correct answer is B.

Assets = Cash/cash equivalents + Invested assets + Use assets (5,000 + 45,000 + 192,000
= 242,000). Liabilities = Short-term debts + Long-term debts (3,000 + 121,000 =
124,000). Assets - Liabilities = Net Worth, so 242,000 - 124,000 = 118,000.

Answer 3 : The correct answer is B.

The income statement describes cash flow for a period of time and contains the
components of salaries and wages, from which expenses are subtracted to determine the
surplus or deficit.

Answer 4: The correct answer is C.

After being informed of the various constraints, implications of their decisions, and other
options they have, the clients can choose to make modifications to their goals.

Answer 5: The correct answer is D.

An investment policy's main purpose is to provide investment guidelines that match the
realities of the client's objectives, investments, and investment markets.

Answer 6: The correct answer is B.

The client has the responsibility of making the investment decision.

Answer 7: The correct answer is A.

Being highly methodical and analytical are attributes of the individualist.

Answer 8: The correct answer is B.

Investment policy should be clearly defined, so that a competent stranger could manage
the portfolio according to the terms of the investment policy statement.

Answer 9: The correct answer is B.

The client must take an active, leading role when the investment policy is being formed.

Financial Planning Academy 99


Answer 10: The correct answer is A.

Because clients are often most strongly attuned to market noise, an advisor adds value by
objectively discriminating between true change and short-term market noise.

Answer 11: The correct answer is A.

Terry is displaying representativeness because he is relying on three years of past


performance to continue in the future.

Answer 12: The correct answer is D.

By purchasing the fund based only on its performance for the last six months, Carol is
putting too much importance on its recent return.

Answer 13: The correct answer is C.

Financial risk is an unsystematic risk that is increased when a firm issues new debt.

Answer 14: The correct answer is A.

Standard deviation measures the variability of a security's return around its average
return.

Answer 15: The correct answer is C.

Rs = 3% + 1.2 (11% - 3%) = 12.6%

Answer 16: The correct answer is B.

Because mid-cap stocks have the lowest positively-correlated asset class of those listed,
they provide more diversification than any other asset class.

Answer 17: The correct answer is C.

The Jensen index compares a portfolio's actual return with the return that could be
expected based on the risk incurred in managing that portfolio. This comparison can be
used to determine if the portfolio did better or worse than expected based on the risk that
was undertaken to attain that return.

Answer 18: The correct answer is D.

A higher Sharpe ratio does indicate better performance based on the risk, as measured by
standard deviation, taken.

Financial Planning Academy 100


Answer 19: The correct answer is C.

In the study, asset allocation policy explained over 93% of the variance of portfolio
returns.

Answer 20: The correct answer is C.

This correct calculation is: (.7 x 4%) + (.3 x 10%) = 5.8%.

Answer 21: The correct answer is B.

Strategic asset allocation attempts to create the optimal balance between expected return
and risk for a long-term horizon.

Answer 22: The correct answer is C.

Current yield is calculated by dividing the annual interest payment by the bond's market
price.

Answer 23: The correct answer is C.

The P/E is correct [20 x 1.20 = 24] and is multiplied, correctly, by next year's earnings
(1.50 x 1.12 = 1.68) to get 40.32.

Answer 24: The correct answer is A.

The late stages of an expansionary period are characterized by high interest rates, high
capacity utilization in the manufacturing sector, increasing wage demands by labor, and
inflation.

Answer 25: The correct answer is B.

All of these elements- goal, method, competencies, and resources- are requirements of an
investment strategy

Answer 26: The correct answer is B.

In the second month, the shares from Month 1 are worth 7,000. The total account value
for Month 2 should be 10,000, so Marjorie needs to buy 3,000 worth of shares; 3,000
divided by the Month 2 price of 7.00 equals 429 shares.

Answer 27: The correct answer is A.

Large cash positions in mutual funds indicate fund managers are bearish; a contrarian
would view this institutional money position as a bullish indicator.

Financial Planning Academy 101


Answer 28: The correct answer is C.

Staggering maturities, which is buying bonds with different but evenly spread maturities
(like 2, 4, 6, 8, and 10 years), is the essence of the ladder strategy.

Answer 29: The correct answer is D.

Unrealistic expectations, an emotional, undisciplined client, and inadequate time horizons


are all enemies of effective strategy.

Answer 30: The correct answer is D.

Growth stocks have high profit margins, carry high betas, and pay small, if any,
dividends.

Answer 31: The correct answer is B.

Total income minus allowed adjustments equals adjusted gross income.

Answer 32: The correct answer is C.

Accelerated depreciation of business assets is a preference item or adjustment for the


alternative minimum tax.

Answer 33: The correct answer is C.

Since payments will cease upon the annuitant's death, the single life option will provide
the highest periodic payment from an annuity.

Answer 34: The correct answer is C.

Life insurance is a poor choice for providing operating funds for a business because
operating funds are needed on an ongoing basis and the cash value of any policy will
eventually be depleted.

Answer 35: The correct answer is D.

Life insurance needs generally decrease as an individual reaches retirement.

Answer 36: The correct answer is B.

The cash requirements would be debt of 25,000 (20,000 + 5,000 the mortgage is excluded
because it will not be paid off), + expenses of 15,000 - liquid assets of 32,000 (30,000 +
2,000) = 8,000.

Financial Planning Academy 102


Answer 37: The correct answer is B.

Mrs. Bartlett owns a renewable five-year term life insurance policy. Such a policy would
have a guarantee to renew provision.

Answer 38: The correct answer is D.

A major reason some life insurance buyers like universal life is that the policy is flexible
in many ways, including its face value, its cash value, and its premiums.

Answer 39: The correct answer is B.

The pressure to meet monthly production expectations and the necessity of doing what is
best for the client can create an ethical conflict.

Answer 40: The correct answer is C.

Even though Treasury bonds have low risk, the representative has a duty to disclose the
risks that Treasury bonds do have.

Answer 41: The correct answer is B.

The advisor should have asked various questions, including the prospective client's
income tax situation, to determine if municipal bonds were suitable before sending the
prospectus

Answer 42: The correct answer is C.

100,000 + 10,000 + 5,000 + 150,000 + 220,000 + 15,000 + 20,000 - 12,000 = 508,000

Answer 43: The correct answer is D.

The yield to maturity on a bond is the total return earned by the bond until it reaches its
maturity date. Accordingly, both the annual interest and any premium or discount of the
bond is factored into its calculation.

Financial Planning Academy 103


Answer 44: The correct answer is C.
The expected rate of return for this portfolio is calculated as follows:
A B C BxC

Description and Percentage Expected


Amount Return

Cash Equivalents 9% 4.5% .40%


20,000

Long-term bonds 26% 7.5% 1.95%


60,000

Common stocks 48% 15.5% 7.44%


110,000

Foreign stocks 40,000 17% 13.0% 2.21%

Total Portfolio 100% 12.00%


230,000

Financial Planning Academy 104


MOCK TEST - II

Question 1: Which of the following are characteristics offered by mutual funds?


I. Professional management
II. Diversification
III. Ready access to shareholders' money
IV. Numerous services such as record keeping, systematic withdrawal plans, and
automatic reinvestment of distributions

A: I and II only
B: I, II, and IV only
C: II, III, and IV only
D: I, II, III, and IV

Question 2 : Which of the following are services typically are offered by mutual funds?
I. Automatic investment
II. Wire / Electronic transfers
III. Systematic withdrawal plans
IV. Telephone redemption

A: I and III only


B: I, II and III only
C: II, III, and IV only
D: I, II, III, and IV

Question 3 : Use the following table to answer the question below.


Total
Share Shares This Shares
Date Transaction Amount Price Transaction Owned

Initial
Jan.2 Purchase 1,000 10.20

Income
Reinvested
Apr.1 .075 11.70

June1 Purchase 500 11.25

Income
Reinvested
June30 .088 12.30

June30 12.30

As of June 30, how many total shares are owned (use three decimal places)?

Financial Planning Academy 105


A: 139.826
B: 141.782
C: 143.111
D: 144.135

Question 4 : Which one of the following is not a risk typically associated with global
equity funds?

A: Purchasing power risk


B: Exchange rate risk
C: Business risk
D: Market risk

Question 5 : The risk/return trade-off relationship generally requires that the investor

A: Assume increasing levels of risk for increased potential return.


B: Forgo current income to maximize capital gains.
C: Assume decreasing levels of risk for increased potential return.
D: Forgo capital gains to maximize current income.

Question 6 : Which of the following are parties to an annuity contract?


I. Contract holder
II. Insurance company
III. Annuitant
IV. Beneficiary

A: I and IV only
B: I, II, and III only
C: I, II, and IV only
D: I, II, III, and IV

Question 7 : Which one of the following is a type of systematic risk?

A: Purchasing power risk


B: Default risk
C: Financial risk
D: Call risk

Question 8 : Which of the following are definitions of risk?

Financial Planning Academy 106


I. The possibility of loss
II. The possibility of not achieving an expected return
III. Variability of returns
IV. Uncertainty of future returns

A: I and II only
B: III and IV only
C: I, II, and IV only
D: I, II, III, and IV

Question 9 : Standard deviation is a measure of

A: The volatility of returns compared to a market index.


B: The degree to which the prices of two securities move together.
C: The dispersion of returns from the investment's mean return.
D: The systematic risk of a security.

Question 10 : What is the approximate change in price of a bond with a duration of 3.35
when interest rates increase 1½%?

A: Increase of 3.35%
B: Increase of 5.03%
C: Decrease of 3.35%
D: Decrease of 5.03%

Question 11 : If a mutual fund has a coefficient of determination (r-squared) of 100, it is


probably a(n)

A: Growth stock fund.


B: Index fund.
C: Bond fund.
D: International stock fund.

Question 12 : The reliability of a mutual fund's beta depends on

A: A low r-squared.
B: The fund's standard deviation.
C: A high correlation between the returns of the fund and the market.
D: The fund's coefficient of variation.

Financial Planning Academy 107


Question 13 : Assuming each of the following bonds has 10 years to maturity, which
one has the least interest rate risk?

A: Treasury zero coupon bond


B: 5% AAA municipal bond
C: 7% AAA corporate bond
D: 4% Treasury note

Question 14 : Which of the following are unsystematic risks?


I. Market risk
II. Business risk
III. Credit risk
IV. Reinvestment rate risk

A: I and III only


B: I and IV only
C: II and III only
D: II and IV only

Question 15 : A mutual fund has an average return of 11% and a standard deviation of
14%. What range of returns would be expected 95% of the time?

A: 0% to 22%
B: -3% to 25%
C: -17% to 39%
D: -31% to 53%

Question 16 : The "real" rate of return is the

A: Actual total return earned.


B: Risk-free rate of return.
C: Nominal rate of return.
D: Nominal rate minus the inflation rate.

Question 17 : A client has a mutual fund that last year had a total return of 12% with a
beta of 1.2. The overall market was up 14% and the risk-free rate of return was 3%. What
was this fund's Jensen index (alpha)?

A: 15.0%
B: 14.4%
C: 2.4%
D: -4.2%

Financial Planning Academy 108


Question 18 : Fund XYZ had a total return of 14%, a beta of .95, and a standard
deviation of 18%. The risk-free return was 4% and the overall market had a total return of
11%. What is this fund's Sharpe index?

A: .55
B: .77
C: 10.53
D: 14.74

Question 19 : The best rate of return to use in judging a portfolio manager's


performance is the

A: Arithmetic average rate of return.


B: Time-weighted rate of return.
C: Geometric rate of return.
D: Rupee-weighted rate of return.

Question 20: A long time horizon (10 years or more) for a client's financial goals is
important because

A: It can convert an otherwise unsuitable investment to a suitable investment.


B: The expected rate of return is more difficult to predict.
C: A portfolio's beta will increase.
D: Periodic market fluctuations take on increased importance.

Question 21: Which one of the following usually is not a part of an investment policy?

A: Performance measurement standards


B: Degree of diversification
C: Specific investments to be purchased
D: Variance of return that the client can tolerate

Question 22: Which one of the following is a characteristic of corporate bonds?

A: Low marketability
B: Moderate variability of cash flows
C: Subject to event risk

Financial Planning Academy 109


Question 23 : When using a computer portfolio optimization program, which of the
following are used as inputs?
I. Standard deviations
II. Covariances
III. Betas
IV. Correlation coefficients

A: I and III only


B: I and IV only
C: II and III only
D: II and IV only

Question 24 : Which of the following are correct regarding an efficient frontier?


I. A point above the efficient frontier is not attainable.
II. If Point A on the efficient frontier is lower and to the left of Point B, which is also
on the efficient frontier, then Point A is superior to Point B.
III. If Point C is on the efficient frontier, then a point directly to the right of Point C,
but below the efficient frontier, has more risk and the same return.
IV. If Point D is below the efficient frontier, that point is attainable but inefficient.

A: I and II only
B: I and IV only
C: I, II, and IV only
D: I, III, and IV only

Question 25 : Which of the following ethical duties or principles of conduct are


incorporated in the asset management process step analyzing information?
I. Disclose
II. Confidentiality
III. Keeping current
IV. Diagnose

A: I and III only


B: I and IV only
C: II and III only
D: II and IV only

Financial Planning Academy 110


Question 26 : T. Prakash is a financial planner who misappropriated funds from a local
charity. By bringing discredit to the financial planning industry, which one of the
following principles of conduct did T. Prakash violate?

A: Diligence
B: Objectivity
C: Professionalism
D: Fairness

Question 27 : The principle of conduct of fairness requires

A: Honesty and candor.


B: Intellectual honesty and impartiality.
C: Service in a prompt and thorough manner.
D: Subordinating one's preferences when balancing conflicts of interest between the
adviser and the client.

Question 28 : George , an investment adviser, has a client wanting current income.


George offers the client a B-rated corporate bond that is paying above average current
income but does not mention the bond's rating. Which one of the following is George
violating?

A: The duty to diagnose


B: The duty to disclose
C: The professionalism principle of conduct
D: The diligence principle of conduct

Question 29: The essence of the duty to keep current is to

A: Understand any changes in the clients' financial situation.


B: Annually update information to appropriate regulatory authorities.
C: Educate oneself regarding new or changing information, laws, and regulations that can
impact clients.
D: Update records of transactions and meetings with clients.

Financial Planning Academy 111


Question 30: Which of the following are strategies that should be employed during the
pre retirement accumulation period?
I. Minimize transaction expenses
II. Minimize taxes on securities held
III. Invest on a regular basis, including during down markets
IV. Start investing for retirement as early as possible

A: I and II only
B: I, II, and III only
C: II, III, and IV only
D: I, II, III, and IV

Question 31: Which of the following are correct regarding selecting a mutual fund?
I. An expense ratio for a stock fund is more important than for a bond fund.
II. Portfolio turnover is an indication of a portfolio manager's investment strategy.
III. A fund with restricted securities adds risk to the portfolio.
IV. In general, it is more advantageous to own a small money market fund than a large
one.

A: I and II only
B: I and III only
C: II and III only
D: II and IV only

Question 32: Which of the following are factors that relate to a client's financial needs
being met?
I. Using time value of money concepts to "run the numbers"
II. Determining if there is a mismatch between the client's current investments and
stated financial goals
III. Assessing if the time frame for the client's goals is reasonable
IV. Judging if tax considerations require portfolio changes

A: I and IV only
B: II and III only
C: I, II, and IV only
D: I, II, III, and IV

Question 33 : Which one of the following is a correctly formulated financial goal?

A: To make 50,000 in the stock market


B: To accumulate adequate retirement funds in 20 years
C: To establish an emergency fund of six months expenses within three years
D: To accumulate 30,000 in four years for a house down payment

Financial Planning Academy 112


Question 34: Which of the following are pertinent to managing a client's expectations?
I. Explaining historic rates of return
II. Explaining the risk component of investing
III. Outlining past performance earned on other clients' accounts
IV. Educating the client that there is no "ideal" investment

A: I and II only
B: III and IV only
C: I, II, and IV only
D: I, II, III, and IV

Question 35 : Which of the following are needed to understand a client's financial


situation?
I. Employer benefits
II. The possibility of being subject to the alternative minimum tax
III. The client's experience with various types of investments
IV. The client's time horizons for financial goals

A: I and III only


B: II and IV only
C: I, II, and III only
D: I, II, III, and IV

Question 36: Adam and Nicobar D,souza provide you with the following information.
Credit card balance 2,250
Mutual funds 77,000
Savings account 9,500
Taxes 18,000
Home 212,750
Mortgage balance 81,200
Vested pension plan 135,000
Credit card payments 4,000

Based on this information, what is the D’souza' net worth?

A: 328,800
B: 332,800
C: 350,800
D: 360,300

Financial Planning Academy 113


Question 37 : What is the general guideline for the basic liquidity ratio?

A: One to three months


B: Three to six months
C: Six to nine months
D: Nine to 12 months

Question 38: Which of the following are factors that influence the amount of cash/cash
equivalents an individual needs?
I. The amount of debt the individual has
II. The types of assets owned
III. The reliability of the individual's income
IV. The timing of significant expenditures

A: I and III only


B: I, II, and IV only
C: II, III, and IV only
D: I, II, III, and IV

Question 39: Which of the following are components of a cash flow statement?
I. Mortgage balance
II. Dividend income
III. Auto loan payments
IV. Taxes

A: I and IV only
B: II and III only
C: II, III, and IV only
D: I, II, III, and IV

Question 40 : Tactical asset allocation

A: Maintains a constant asset mix.


B: Shifts the portfolio asset mix in response to anticipated changes in the market.
C: Alternates between risky types of investments and "safe" assets.
D: Is generally employed by only institutional investors.

Financial Planning Academy 114


Question 41 : If the weighting of the asset classes within a portfolio has changed due to
performance differences of the assets, the portfolio's

A: Risk has increased.


B: Expected return has decreased.
C: Holding period has changed.
D: Risk/return relationship has changed.

Question 42 : Which of the following can change the efficient frontier?


I. Adding asset classes
II. Decreasing the level of cash holdings
III. Increasing the client's risk tolerance level
IV. Changing the expected returns of assets

A: I and IV only
B: I, II, and III only
C: II, III, and IV only
D: I, II, III, and IV

Question 43 : Which of the following are correct regarding an efficient frontier?


I. A point above the efficient frontier is not attainable.
II. If Point A on the efficient frontier is lower and to the left of Point B, which is also
on the efficient frontier, then Point A is superior to Point B.
III. If Point C is on the efficient frontier, then a point directly to the right of Point C,
but below the efficient frontier, has more risk and the same return.
IV. If Point D is below the efficient frontier, that point is attainable but inefficient.

A: I and II only
B: I and IV only
C: I, II, and IV only
D: I, III, and IV only

Question 44 : Which of the following are components of invested assets on the statement
of financial position?
I. Money market funds
II. Cash value of life insurance
III. Individual retirement accounts
IV. Vested portion of a pension fund

A: I and II only
B: III and IV only
C: I, II, and III only
D: II, III, and IV only

Financial Planning Academy 115


Question 45 : An investment policy should not incorporate

A: The acceptable risk tolerance level.


B: The investment strategies that are acceptable.
C: An approximation of asset allocation among acceptable asset classes.
D: The acceptable amount of gains from different asset classes.

Question 46: An asset allocation strategy that centers on creating a fixed percentage of
assets is called

A: Core/satellite asset allocation.


B: Strategic asset allocation.
C: Tactical asset allocation.
D: Dynamic asset allocation.

Question 47:
Assume you have the following values for Portfolio A:
Rate of return = 14%
Beta = 1.1
Standard deviation = 0.10
In addition, the market rate of return is 12% and the risk-free rate is 5%. Based on this
information, which one of the following statements is correct?

A: The Treynor performance index is 0.9.


B: The Sharpe index is 0.08.
C: The Jensen index is -2.9%.
D: The Jensen index is 1.3%.

Financial Planning Academy 116


Solution : - ( Mock Test – II )

Answer 1: The correct answer is D.

Mutual funds offer shareholders professional management, diversification of securities,


quick access to their money through redemptions within seven days, and numerous
services such as record keeping, systematic withdrawal plans, and automatic reinvestment
of distributions.

Answer 2: The correct answer is D.

Mutual funds typically offer numerous services, including automatic investment, wire
transfers, systematic withdrawal plans, and telephone redemption.

Answer 3: The correct answer is D.

The initial purchase acquires 98.039 shares (1,000 divided by 10.20). The 98.039 shares
times the .075 income distribution per share equals 7.353 (mutual fund statements often
compute to three decimal places). 7.353 divided by the price of 11.70 buys .628 shares.
This is added to 98.039 shares to equal 98.667 shares as of April 1. Continuing in the
same way, the total shares at June 1 are 143.111 and at June 30 are 144.135.

Answer 4: The correct answer is A.

Purchasing power risk is associated with bond funds because of bonds' fixed interest and
principal payments. Global equity funds have exchange rate risk attached to their foreign
stocks, and, as with any stocks, business and market risk.

Answer 5: The correct answer is A.

The risk/return relationship is based on the fact that in order to get a higher return, an
investor must take a higher risk. Likewise, when taking lower risk, the investor should
expect a lower return. This relationship does not require that either current income or
capital gains must be given up.

Answer 6: The correct answer is D.

In an annuity contract, the contract holder owns the policy, the insurance company issues
the annuity, the annuitant receives the annuity payments, and the beneficiary receives any
proceeds due from the annuity upon the death of the contract holder or annuitant.

Answer 7: The correct answer is A.

Purchasing power risk is a systematic (nondiversifiable) risk, but default risk, financial
risk, and call risk are unsystematic risks.

Financial Planning Academy 117


Answer 8: The correct answer is D.

The possibility of loss or not achieving an expected return, the variability of returns, and
the uncertainty of future returns are all definitions of risks.

Answer 9: The correct answer is C.

Standard deviation is a measure of total risk and measures the dispersion of returns from
the investment's mean, or average, return.

Answer 10: The correct answer is D.

The approximate price change of this bond is 3.35 × 1.5% = 5.03%. If interest rates
increase, this bond will decrease about 5.03%.

Answer 11: The correct answer is B.

A mutual fund with a coefficient of determination of 100 means that 100% of the
variance in the mutual fund's rate of return can be explained by the variance in the
market's rate of return. Therefore it is probably an index fund.

Answer 12: The correct answer is C.

The reliability of a mutual fund's beta is dependent upon a high correlation (r, not r-
squared) between the returns of the fund and the market. The fund's standard deviation is
part of the beta calculation but by itself does not affect the beta's reliability. The
coefficient of variation is the fund's standard deviation divided by its average return

Answer 13: The correct answer is C.

If the maturity is equal and the quality is essentially equal, the bond with the highest
coupon rate will have the least interest rate risk.

Answer 14: The correct answer is C.

Unsystematic risk is the risk that is unique to an individual investment, so business risk
and credit risk are the unsystematic risks of those listed in the question.

Answer 15: The correct answer is C.

In this situation, 95% of the time this fund's return is expected to fall within two standard
deviations. Therefore, 11% plus and minus 28% (2 × 14%) results in a range of -17%
(11% - 28%) and 39% (11% + 28%).

Financial Planning Academy 118


Answer 16: The correct answer is D.

The real rate of return is the nominal, or stated, rate of return less the inflation rate.

Answer 17: The correct answer is D.

The Jensen index for this fund is 12% - [3% + 1.2(14% - 3%)] = 12% - [3% + 13.2%] =
12% - 16.2% = -4.2%.

Answer 18: The correct answer is A.

The Sharpe index for this fund is (14% - 4%)/18% = .55

Answer 19: The correct answer is B.

The time-weighted rate of return is the best measure of a portfolio manager's performance
because it isolates the performance achieved and negates the effects of contributions and
withdrawals of investors, a factor that is beyond the manager's control.

Answer 20: The correct answer is A.

With a long time horizon, an otherwise unsuitable investment can become a suitable
investment. For example, common stocks are inappropriate for a short-term goal since a
bear market can reduce their market value to below what is needed to achieve the goal.
For a goal having a long time horizon, common stocks are appropriate since the periodic
market fluctuations take on less importance as up markets offset down markets. In
addition, the longer the time frame, the more predictable the expected rate of return is
because a given asset tends to earn its long-term rate of return. Time does not necessarily
increase a portfolio's beta.

Answer 21: The correct answer is C.

An investment policy typically states the types of investments to be purchased but not the
specific investments to be purchased.: Which one of the following usually is not a part of
an investment policy?

Answer 22: The correct answer is C.

Corporate bonds generally have high marketability, low variability of cash flows, do not
have a minimum required investment (although new issues are generally in
denominations of 1,000), and are subject to event risk (the possibility of an unexpected
and negative occurrence of some type, such as disclosure of major corporate fraud).

Financial Planning Academy 119


Answer 23: The correct answer is B.

A computer portfolio optimization uses as inputs for each asset standard deviations and
correlation coefficients. In addition, expected returns are also inputs.

Answer 24: The correct answer is D.

Points above the efficient frontier are not attainable. Points below the efficient frontier
are attainable but inefficient. All points on the efficient frontier represent the best
risk/return tradeoffs and therefore none of them are superior to any of the others. A point
directly to the right and below a point on the efficient frontier has more risk but the same
return.

Answer 25: The correct answer is D.

The step of analyzing information involves the confidentiality principle of conduct and
the duty to diagnose.

Answer 26: The correct answer is C.

Professionalism includes the obligation to enhance and maintain the profession's public
image.

Answer 27: The correct answer is D.

The principle of conduct of fairness requires subordinating one's feelings, prejudices, and
desires when balancing conflicts of interest between the firm, the adviser, and the client.
Honesty and candor is part of the integrity principle of conduct. Intellectual honesty and
impartiality is required by the principle of objectivity. Diligence requires providing
service in a prompt and thorough manner.

Answer 28: The correct answer is B.

The duty to disclose requires the investment professional to disclose, among other things,
the risks of any investment recommended.

Answer 29: The correct answer is C.

The duty to keep current involves understanding new, and/or changes in, products,
financial planning strategies, laws, and regulations that might impact clients' financial
plans.

Financial Planning Academy 120


Answer 30: The correct answer is D.

Minimizing transaction expenses and taxes, investing on a regular basis (especially


during down markets), and starting to invest for retirement as soon as possible are all
strategies that should be used during the pre retirement accumulation phase.

Answer 31: The correct answer is C.

Portfolio turnover is an indication of a portfolio manager's strategy because it indicates,


for example, if the manager tries to catch quick turns in the market or buys undervalued
stocks and holds them until full valuation. Restricted securities add risk because they
cannot be sold until a certain time and may be difficult to price. An expense ratio is more
important for a bond fund than a stock fund because the returns from a bond fund are
usually lower. A large money market fund can trade in larger units, thereby getting better
prices, and can pass on economies of scale to its shareholders.

Answer 32: The correct answer is D.

There are numerous factors that relate to a client's financial needs being met. Time value
of money concepts can be used to determine if a goal is financially achievable. In
addition, there can be a mismatch between a client's current investments and those goals.
The time frame and tax considerations are also both important relative to the financial
needs of the client.

Answer 33: The correct answer is D.

This goal is specific in dollar amounts, time frame, and objective.

Answer 34: The correct answer is C.


Managing a client's expectations is done by educating the client about historic rates of
return, the risks of investing, and that there is no "ideal" investment, among other things.
The past performance earned on other client's accounts is not useful in managing
expectations because that performance pertains to other clients who may have had
different goals and risk/return profiles than the client.

Answer 35: The correct answer is D.

There are numerous considerations that an investment professional needs to know about a
client, including his or her employer benefits, experience with various investments, the
client's time horizons for financial goals, and the possibility of being subject to the
alternative minimum tax.

Financial Planning Academy 121


Answer 36: The correct answer is C.

Assets are 77,000 + 9,500 + 212,750 + 135,000 = 434,250. Liabilities are 2,250 + 81,200
= 83,450. Therefore, net worth is 434,250 - 83,450 = 350,800. Taxes and credit card
payments are cash flow statement outflows.

Answer 37: The correct answer is B.

Although the needs for liquidity for individuals differ, the general rule for the basic
liquidity ratio is three to six months.

Answer 38: The correct answer is D.

The amount of cash/cash equivalents needed by an individual is based on several factors,


including the amount of debt and types of assets owned by the individual, the reliability
of the person's income, and the timing of significant expenditures.

Answer 39: The correct answer is C.

Dividend income, auto loan payments, and taxes are all cash flow items. A mortgage
balance is a liability on the statement of financial position (although mortgage payments
would be on the cash flow statement).

Answer 40: The correct answer is B.

Tactical asset allocation, used by both individuals and institutions, tries to beat market
returns by buying assets anticipated to outperform in the near future and selling assets
expected to underperform. Strategic asset allocation keeps a constant asset mix, and
dynamic asset allocation, usually employed by institutional investors when it is used,
shifts between risky and "safe" investments.

Answer 41: The correct answer is D.

When the weightings (percentages) of assets within a portfolio change, the portfolio's
risk/return relationship changes. Either risk or return can increase or decrease, depending
on the changes in the percentages of the asset classes affected.

Answer 42: The correct answer is D.

Many factors can change the efficient frontier including adding asset classes and
changing the level of cash holdings, changing the client's risk tolerance level, and
changing the expected returns of assets.

Financial Planning Academy 122


Answer 43: The correct answer is D.

Points above the efficient frontier are not attainable. Points below the efficient frontier
are attainable but inefficient. All points on the efficient frontier represent the best
risk/return tradeoffs and therefore none of them are superior to any of the others. A point
directly to the right and below a point on the efficient frontier has more risk but the same
return.

Answer 44: The correct answer is B.

Individual retirement accounts and the vested portion of a retirement fund are invested
assets. A money market fund and the cash value of a life insurance policy (because it can
be borrowed) are classified as cash equivalents

Answer 45: The correct answer is D.

An investment policy should not incorporate the acceptable amount of gains from
different asset classes because the policy presents broad guidelines and addresses the
entire portfolio, not specific investment results from certain asset classes.

Answer 46: The correct answer is B.

Strategic asset allocation tries to maintain a fixed percentage in the asset classes. Tactical
asset allocation changes the asset mix with changes in market conditions. Dynamic asset
allocation involves moving between risky and risk less assets depending on market
conditions. Core/satellite asset allocation divides a portfolio into two parts: (1) 70%-80%
in index funds or ETFs, and (2) the remainder of the portfolio in actively managed
investments, such as commodity funds and sector funds.

Answer 47: The correct answer is D.

The Jensen index is 0.14 - [0.05 + 1.1(0.12 - 0.05)] = 0.14 - [0.05 + 0.077] = 0.14 - 0.013
= 1.3%. The Treynor index is (0.14 - 0.05)/1.1 = 0.08. The Sharpe index is (0.14 -
0.05)/.1 = 0.9.

Financial Planning Academy 123


Mock Test - III

Question 1: A debt service ratio below 35% is one indicator of financial health. What is
the formula used to calculate a person's debt service ratio?
A: Total debt divided by total assets
B: Annual debt repayments divided by annual take-home pay
C: Annual debt repayments divided by gross income
D: Investment assets divided by net worth

Question 2: What are the two primary considerations in determining a person's ability to
handle debt?
A: The cost of the item and the monthly payments
B: The person's total debt load and the monthly debt service
C: The person's annual income and the number of creditors
D: The person's net worth and the total debt load

Question 3: Money for small, unplanned expenses would be found in which one of the
following budget categories?

A: Housing maintenance and repair


B: Miscellaneous expenses
C: Reserve funds
D: Investments

Question 4: What influence does the rate-of-return assumption have on goal planning?
I. A high rate-of-return assumption may result in fewer dollars being invested to
achieve a particular goal.
II. A high rate-of-return assumption will allow greater flexibility in achieving a
particular goal.
III. A low rate-of-return assumption will create an apparent need for larger
investments.
IV. A low rate-of-return assumption will more nearly guarantee that a particular goal
will be achieved.
A: I and II only
B: I and III only
C: I and IV only
D: II and III only

Question 5: In 10 years, Ali will receive 1 million from a trust fund. If the fund is
earning 11% annual interest, compounded monthly, how much is the fund worth now?
A: 334,543
B: 352,184
C: 485,414
D: 912,790

Financial Planning Academy 124


Question 6: Does the future value of a present sum increase or decrease if the compound
rate is increased, and why?

A: It decreases, because the compounding process begins with the first payment and
includes the final payment.
B: It decreases, because a smaller present value is required when the interest earned on
interest and principal combined is higher.
C: It increases, because as the compounding rate is increased, higher interest is earned on
principal and previous interest over the term of the investment.
D: It increases, because interest is compounded one time more than it is with an ordinary
annuity because of the timing of payments.

Question 7: Which one of the following lists the two main parts of total investment risk?

A: unsystematic and systematic


B: systematic and market
C: systematic and nondiversifiable
D: unsystematic and diversifiable

Question 8:Which of the following items identify the risk, marketability, and liquidity
profile of Treasury notes and bonds?
I. Low liquidity and low marketability
II. Moderate liquidity and high marketability
III. Purchasing power risk and interest rate risk
IV. Market risk and business risk

A: I and III only


B: I and IV only
C: II and III only
D: II and IV only

Question 9: Which one of the following best defines standard deviation?

A: the way in which the prices of two securities move in relation to each other
B: a measure of how much an investment's return deviates from its average return over a
period of time
C: a security's volatility (based on historical performance data) relative to a benchmark
market
D: a reflection of individual factors affecting specific companies or investments as
opposed to the market as a whole

Financial Planning Academy 125


Question 10: Which of the following questions characterize the basic asset allocation
process?
I. Which types of investments should be included or excluded?
II. How much weight should be given to each asset class?
III. Which investment vehicles should be used?
IV. Which investment vehicles have the highest long-term return?

A: I and II only
B: III, and IV only
C: I, III, and IV only
D: II, III, and IV only

Question 11: Which of the following factors normally are evaluated when doing
technical analysis?
I. Industry
II. Market averages
III. Trading volume
IV. Advances/declines

A: I and III only


B: I and IV only
C: I, III, and IV only
D: II, III, and IV only

Question 12: Which of the following are rights normally (or frequently) held by common
stock holders?
I. The right to receive declared dividends
II. The right to elect members to the board of directors
III. The right to purchase enough of any newly issued shares of stock to protect their
percentage of ownership
IV. The right to make a detailed inspection of all the corporation's account books

A: I and II only
B: I, II, and III only
C: I, II, and IV only
D: II, III, and IV only

Question 13: If an investor expects the price of a stock to increase, which one of the
following will generate the highest return?
A: shorting the stock
B: going long
C: buying the stock on margin
D: selling short against the box

Financial Planning Academy 126


Question 14: Short positions are riskier than long positions because

A: they require a larger investment than long positions.


B: short positions are more difficult to close out.
C: there is less marketability with short positions.
D: an investor can lose more than his or her original investment with a short position.

Question 15: Which of the following factors are normally evaluated when doing
fundamental analysis?
I. Interest rates, gross domestic product, inflation, unemployment, and inventories
II. Market activity and overall market trends
III. Past records of assets, earnings, sales, products, management, and markets
IV. Balance sheets and income statements of companies

A: I, II, and III only


B: I, II, and IV only
C: I, III, and IV only
D: I, II, III, and IV

Question 16: Which of the following are the two major risks of common stock?
I. Business risk
II. Interest rate risk
III. Purchasing power risk
IV. Market risk

A: I and II only
B: I and IV only
C: II and III only
D: III and IV only

Question 17: Which one of the following best describes the treatment of dividends for
participating preferred stock?

A: It pays fixed dividends and a portion of company profits that remains after all other
dividends are paid.
B: Its cumulative dividends accumulate and must be paid before common stock
dividends.
C: It pays dividends at a specified rate and has a higher priority than common stock.
D: It pays regular dividends and usually comes with voting rights.

Financial Planning Academy 127


Question 18: Which one of the following is a characteristic of preferred stock?

A: Their dividend rates are usually fixed.


B: The stockholders normally have voting rights.
C: They typically have fixed maturities.
D: The dividends are usually paid from before-tax income.

Question 19: The face, or par, value of a bond is

A: the amount paid by the issuer when the bond matures.


B: its price in the secondary market.
C: the amount paid by the investor when the bond is purchased.
D: its anticipated yield to maturity.

Question 20: Which of the following items identify the risk, marketability, and liquidity
profile of high-grade municipal bonds?
I. Purchasing power and interest rate risk
II. Business, financial, interest rate, and default risk
III. High liquidity, low marketability
IV. Moderate liquidity, high marketability

A: I and III only


B: I and IV only
C: II and III only
D: II and IV only

Question 21: The "load" associated with mutual funds may best be described as

A: the portfolio composite.


B: an advisory fee.
C: the expected rate of return.
D: a sales charge.

Question 22: Which one of the following is a characteristic of growth mutual funds?

A: The primary objective is capital appreciation.


B: The primary objective is current income.
C: The fund invests primarily in fixed-income investments.
D: The fund invests primarily in other mutual funds.

Financial Planning Academy 128


Question 23: Which of the following assumptions must be made in doing the
calculations for forecasting retirement income needs?
I. Rate of inflation
II. Number of years during which the client will need income
III. Inadequate money supplies and the breakdown of personal property
IV. After-tax rate of return on investments

A: I, II, and III only


B: I, II, and IV only
C: I, III, and IV only
D: II, III, and IV only

Question 24: Which one of the following is not a common reason to delay saving for
retirement?

A: procrastination
B: fear of inflation
C: paying off debt
D: confusion about how to begin

Question 25: Which of the following are techniques for managing risk?
I. Avoidance
II. Sharing
III. Reduction
IV. Retention

A: I, II, and III only


B: I, III, and IV only
C: II, III, and IV only
D: I, II, III, and IV

Question 26: Which of the following are basic rules of risk management?
I. Do not risk more than you can afford to lose
II. Consider the odds
III. Transfer only those risks that are likely to increase in expense
IV. Do not risk a lot for a little

A: I and II only
B: I and IV only
C: I, II, and IV only
D: I, III, and IV only

Financial Planning Academy 129


Question 27: From the point of view of an insurance company, which of the following
are requirements for an insurable risk?
I. The loss must be catastrophic
II. The law of large numbers must apply
III. The loss must be accidental or fortuitous
IV. The loss must be definite and measurable

A: I and III only


B: I, II, and III only
C: I, III, and IV only
D: II, III, and IV only

Question 28: Which insurance principle exists so that a person does not profit from his
or her loss?

A: principle of indemnity
B: principle of insurable interest
C: principle of adhesion
D: principle of aleatory contracts

Question 29: Term life insurance provides which one of the following benefits?

A: It has fixed, level premiums throughout the entire contract period.


B: It builds cash values, which, while designed to help keep the premium level, can be
used by the insured.
C: It provides insurance coverage during a specified period.
D: It covers two or more lives and can be either a first-to-die or second-to-die policy.

Question 30: The primary purpose of major medical insurance is to do which one of the
following?

A: Provide coverage for the large medical expenses that could become financially
damaging if the insured suffers a serious injury or long-term illness.
B: Provide hospital services rather than cash payments for a certain number of days.
C: Pay for the costs of surgical procedures and nonsurgical doctor's fees.
D: Reimburse the patient for some or all hospital expenses.

Financial Planning Academy 130


Solution :- ( Mock Test - III )

Answer 1: The correct answer is B.

The debt service ratio formula is annual debt repayments divided by annual take-home
pay.

Answer 2: The correct answer is B.

The two primary considerations in determining a person's ability to handle debt are the
person's total debt and the monthly debt service. Annual income and net worth are
secondary considerations because if individuals overspend, they may not be able to
handle their debt.

Answer 3: The correct answer is C.

A budget item called reserves is used for small, unplanned expenses. The category of
miscellaneous expenses is used for items that don't fit into other categories but are
expected expenses.

Answer 4: The correct answer is B.

If a high rate of return is assumed, fewer dollars are needed to achieve a goal, but that
will not necessarily allow greater flexibility in achieving a goal because fewer
investments offer the possibility of achieving a high rate of return. A low rate of return
will not necessarily mean a goal will be achieved because, for example, if the time frame
for the goal is long, a low return might mean not achieving the goal. It will, however,
increase the likelihood of needing larger investments to achieve a goal.

For monthly compounding, if the calculator is set at 12 pmt/year, 1,000,000 = FV, 11 =


I/YR, 10 × 12 = 120 = N, then PV = 334, 543. If the calculator is set at 1 pmt/year,
1,000,000 = FV, 11/12 = .917 = I/YR, 10 × 12 = 120 = N, then PV = 334,543.

Answer 6: The correct answer is C.

A present value will always increase to a higher future value the more often compounding
increases because interest is earned on interest more frequently.

Answer 7: The correct answer is A.

Total investment risk is unsystematic (diversifiable) risk plus systematic


(nondiversifiable) risk. Market risk is one type of systematic risk.

Financial Planning Academy 131


Answer 8: The correct answer is C.

The major risks of Treasury notes and bonds, like all fixed-income securities, are
purchasing power risk and interest rate risk. There is a large market for them so they have
high marketability, and they generally fluctuate moderately from their purchase price and
therefore have moderate liquidity as defined in this course.

Answer 9: The correct answer is B.

Standard deviation measures the amount by which an investment's return fluctuates from
its average return. The degree to which securities prices move together is correlation. A
security's volatility relative to a benchmark index is measured by beta. Individual factors
affecting an investment is market risk.

Answer 10: The correct answer is A.

The basic asset allocation process involves determining which assets to use and the
weights to be given to each asset class. The specific investment vehicles are considered
after determining what assets to include

Answer 11: The correct answer is D.

Technical analysis involves tracking prices and volume so market averages, trading
volume, and advances/declines are used. Industry analysis is done in fundamental
analysis.

Answer 12: The correct answer is B.

An investor in common stock has several rights, including receiving dividends declared
by the board of directors, electing members of the board, and purchasing new shares
issued through preemptive rights. Shareholders have the right to receive financial reports
but not to inspect the books of the corporation (this is done by auditors).

Answer 13: The correct answer is C.

Selling short, or shorting a stock, is done in anticipation of the stock falling in price.
Going long, or buying the stock, is done in anticipation of the stock increasing in price,
but buying the stock on margin, where part of the purchase price is borrowed, will result
in a higher return if the stock goes up. For example, if an investor buys 1,000 worth of
stock and sells it for 1,500, the investor made 500. However, if the investor invests 500
and borrows 500 for the purchase, he or she will make 1,000 (1,500 - 500) on the amount
invested.

Financial Planning Academy 132


Answer 14: The correct answer is D.

Selling short, or shorting a stock, is selling the stock in anticipation of it falling in price,
at which time the stock is bought to close out the transaction. If, instead, the stock goes
up, it can go up beyond the investor's original investment. Short positions do not
necessarily require a larger investment than long positions, and they can be closed out
relatively easily.

Answer 15: The correct answer is C.

Fundamental analysis involves examination of the overall economy, characteristics of


industries, and specifics of the company such as balance sheets, income statements,
management, products, markets, etc. Market activity and market trends are part of
technical analysis.

Answer 16: The correct answer is B.

The two major risks of common stock are business risk and market risk. Interest rate risk
and purchasing power risk are the two major risks of fixed-income securities.

Answer 17: The correct answer is A.

Preferred stock generally pays a fixed dividend. If it is participating, then the dividend
increases from a portion of the company profits left after other dividends are paid.

Answer 18: The correct answer is A.

Usually, preferred stock pays a fixed dividend from after-tax income. Unlike bonds, it
does not have a maturity. It usually does not have voting rights.

Answer 19: The correct answer is A.

A bond's face, or par, value is the dollar amount the issuer will pay the investor at
maturity. The price the bond is trading at currently in the secondary market is its current
price. The amount paid at time of purchase is its purchase price. The yield to maturity is
the bond's total return expressed as a percentage.

Answer 20: The correct answer is B.

The major risks of any bond, including municipal bonds, are interest rate risk and
purchasing power risk. High-grade municipal bonds generally have an active market so
they have high marketability. They usually have some, but not major, fluctuations in price
so they have moderate liquidity as defined in this course.

Financial Planning Academy 133


Answer 21: The correct answer is D.

A mutual fund load, by definition, is a sales charge. An advisory fee is part of a fund's
annual operating expenses.

Answer 22: The correct answer is A.

Another term for growth is capital appreciation. Growth funds attempt to achieve the
objective of growth or capital appreciation by investing in common stocks that are
expected to increase in price.

Answer 23: The correct answer is B.

Once an individual determines the additional amount he or she needs to retire today
(amount needed less income available for retirement), forecasting his or her retirement
income need-- and the amount needed to provide it-- becomes a time value of money
problem. This requires three assumptions: (1) a rate of inflation, (2) an after-tax rate of
return on your investments, and (3) the number of years during which you will need
income. Money supplies and personal property are taken into account in determining the
amount an individual needs in order to retire today.

Answer 24: The correct answer is B.

Procrastination, paying off a debt, and confusion about how to begin are all common
reasons why people delay saving for retirement. Fear of inflation motivates some people
to begin saving for retirement.

Answer 25: The correct answer is D.

The five essential techniques for managing risk are risk transfer, risk avoidance, risk
reduction, risk sharing, and risk retention.

Answer 26: The correct answer is C.

Mehr and Hedges came up with the following three rules of risk management:
Don't risk more than you can afford to lose
Consider the odds
Don't risk a lot for a little

Financial Planning Academy 134


Answer 27: The correct answer is D.

From the insurance company's point of view, an insurable risk must meet four
requirements:
the law of large numbers
the loss must be accidental or fortuitous
the loss must be definite and measurable
the loss must not be catastrophic

Answer 28: The correct answer is A.

The principle of insurable interest must exist for both life and property insurance. With
life insurance the purchaser and beneficiary of a life insurance policy must have an
insurable interest in the insured when the policy is first purchased. With property
insurance the insurable interest must exist at the tine of the claim. Without these
limitations, policyowners could profit from the death of an insured person or the
destruction of the insured property. The outcome of the contract is dependent on chance
and the dollars exchanged are of substantially unequal amounts. Additionally, one party
writes the contact and the other party has no say in the matter. The principle of indemnity
is based on the idea that when persons suffer a loss, they should be made whole but not
profit from the loss.

Answer 29: The correct answer is C.

The simplest and perhaps purest form of life insurance is term life insurance. Term
insurance offers pure protection for an individual and does not build up cash value. The
policy will pay the face amount of the contract to the listed beneficiary if the insured dies
within a given coverage time. With annually renewable term insurance, the premium or
cost for the coverage increases each year.

Answer 30: The correct answer is A.

Major medical insurance provides coverage for the large medical expenses associated
with a serious injury or long-term illness that could be financially damaging.

Financial Planning Academy 135


Mock Test - IV

Question 1.
Which of the following is NOT an example of government monetary policy to affect
interest rates?

A Buying and selling government securities in the interbank market via the central bank.
B Intervening in the foreign exchange market.
C Adjusting its expenditure and taxation policy, thus affecting the supply of and demand
for funds.
D Imposing strict controls on the lending operations of financial institutions.

Question 2.
Which of the following are NOT COMMON functions of a bank?

I) Facilitating the mobilisation of both wholesale and retail funds.


II) Prudential supervision of fellow banks.
III) Enabling investors to save money in order to create wealth.
IV) Implementing the government’s fiscal policy.

A I) and II) only.


B I) and III) only.
C II) and IV) only.
D III) and IV) only.

Question 3.
Which of the following is MOST LIKELY to be classified as a “direct control approach”
in monetary policy?

A An increase in the level of direct taxes in the budget announcement.


B Trading of government securities in the interbank market between the central bank and
licensed banks authorised to trade such securities by the government.
C A government’s appointment of its finance chief.
D Controlling interest rates on loans.

Question 4.
Which of the following is NOT a method of raising equity finance?

A Private placements.
B Equity charters.
C Initial Public Offerings.
D Dividend reinvestment.

Financial Planning Academy 136


Question 5.
Paul has 800 shares in CBZ Ltd. before a 1 for 4 bonus issue is announced. If the shares
were traded at BSE 2.00 immediately before the bonus issue, what is the price of Paul’s
shares immediately after the bonus issue?
A 1.20
B 1.40
C 1.60
D 1.80

Question 6.
SITA is in his late 30s. In order to meet his financial goals in the next 30 years, Sita has
asked RAM to design an investment scheme for her. Which of the following financial
service personnel would Ram MOST LIKELY be?
A Corporate financier.
B Fund manager.
C Stockbroker.
D Professional financial planner.

Question 7.
Which of the following are essential procedures involved in the process of creating a
financial plan by a Professional financial planner?

I) Analyse the client’s financial status and risk tolerance.


II) Compile a financial plan that meets the client’s objectives.
III) Provide margin financing for the client to implement stock investments according to
the financial plan.
IV) Monitor and review the performance of the client’s stock investments and revise
investment strategies as needed.

A I) and II) only.


B III) and IV) only.
C I), II) and IV) only.
D II), III) and IV) only.

Financial Planning Academy 137


Question 8.
Mr. Himesh has inherited a large amount of wealth from his parents. He has decided to
invest a portion of his wealth in managed funds. Which of the following is NOT a valid
reason for Mr. Himesh to invest in managed funds?

A To gain access to professional investment management services.


B To increase the exposure of his portfolio to a wider range of investment instruments.
C To have a tailor-made investment portfolio designed exclusively for him by a fund
manager.
D To gain access to markets which are not directly accessible to retail investors in the
stock market.

Question 9.
Which of the following transactions utilises risk management techniques?

I) The purchase of financial instruments to hedge an existing portfolio.


II) The sale of options to reduce exposure to a particular industry.
III) Increasing receivables through an increase in inventory turnover.
IV) Exchanging your fixed interest payable for a counter-party’s floating interest payable.

A I), II) and III) only.


B I), II) and IV) only.
C I), III) and IV) only.
D II), III) and IV) only.

Question 10.
Jason is a fund manager and forecasts that the recent high inflation rate will induce the
central bank to post an interest rate hike. The portfolio he is currently managing has a
weighting of 20% in floating-rate debt securities, 40% in fixed-rate debt securities and
40% in equity securities. Based on his forecast, which of the following actions would be
MOST APPROPRIATE for Jason to take?

A Inform the investors of his fund and seek their advice on the most suitable strategy.
B Unwind his portfolio and wait for the appropriate time to invest the cash on hand in
these markets again with the same weighting.
C Increase investment in equity securities of companies which have large amounts of
outstanding loans on their balance sheets, and reduce the weighting of debt securities in
his portfolio.
D Reduce his investment in both fixed-rate debt securities and equity securities while
increasing his investment in floating-rate debt securities.

Financial Planning Academy 138


Question 11.
After Zubin has purchased a portfolio of short-term debt securities, the prices of the debt
securities have dropped significantly due to a major increase in interest rates
implemented by the government. To which type of risk is Zubin’s portfolio exposed?

A Market risk.
B Credit risk.
C Strategic risk.
D Operational risk.

Question 12.
Which of the following activities utilise the advantages of derivatives?

I) A fund manager buys call options to minimise the impact of the expected short-term
market fluctuation on his portfolio.
II) A trader purchases spot gold contracts and immediately transacts an opposite trade of
an equal amount in the futures market.
III) A corporation redeems cash from a managed fund in order to increase the liquidity of
its pension portfolio.
IV) An investor uses futures contracts to magnify the size of his portfolio.

A I) and III) only.


B III) and IV) only.
C I), II) and III) only.
D I), II) and IV) only.

Question 13.
It is now December. Yash Raj Limited expects to receive 1 million at the end of February
for sales of Diamond Export. The company plans to invest all receivables in fixed-interest
securities. However, it is expected that interest rates will fall in January. Which of the
following is the MOST APPROPRIATE strategy for Yash Raj to adopt in order to
guarantee the return of its future investment at the current interest rate level?

A Enter into a fixed to floating interest rate swap.


B Enter into a floating to fixed interest rate swap.
C Enter into a forward rate agreement.
D Do nothing because falling interest rates will benefit its investment.

Financial Planning Academy 139


Question 14.
What is a bond generally called when its coupon rate is set to vary against a
predetermined benchmark rate?

A Floating rate bond.


B Fixed rate bond.
C Benchmarked coupon bond.
D Predetermined coupon bond.

Question 15.
An investor finds a newly issued bond offering a slightly higher yield than the risk-free
rate. Which of the following statements is MOST APPROPRIATE in describing this
newly issued bond?

A There is no risk premium applied to the bond.


B The issuer has a very high credit rating.
C The market anticipates that interest rates will rise slightly in the near future.

Question 16.
While plotting the yield curves of selected debt securities of the same type, Kairav finds
that the yields of short-term securities are generally higher than the yields of longer term
securities. Which type of yield curve does this situation illustrate?

A A positive yield curve.


B An inflationary yield curve.
C A deflationary yield curve.
D An inverse yield curve.

Question 17.
Which of the following are NOT COMMON characteristics of both rights issues and
bonus issues?

I) New shares are issued to existing shareholders.


II) Free shares are issued to existing shareholders.
III) A source of funds for the underlying company.
IV) The shareholding of non-subscribing shareholders will be diluted.

A I) and II) only.


B III) and IV) only.
C I), III) and IV) only.
D II), III) and IV) only.

Financial Planning Academy 140


Question 18.
After several years of economic recession, Country X is now facing a severe budget
deficit. The government of Country X decides to reduce government spending and to
increase the income tax rate in order to balance its budget. What is the LIKELY impact
on the economy upon implementation of this policy?

A The economy will be worse off as the result of a contractionary fiscal policy.
B The economy will be worse off as the result of a contractionary monetary policy.
C The economy will be improved as the result of an expansionary fiscal policy.
D The economy will be improved as the result of an expansionary monetary policy.

Question 19.
Four stocks within the same industry have the following performance characteristics:
Stock Expected return Standard deviation

Return Standard Deviation


I 5% 0.25
II 5% 0.30
III 8% 0.25
IV 8% 0.30

Which ONE of the stocks provides the BEST investment opportunities for a rational
investor?

A Stock I.
B Stock II.
C Stock III.
D Stock IV.

Financial Planning Academy 141


Question 20.
Of the four pairs of portfolios, which pair provides the highest level of diversification?

a) Portfolio 1 & 2: with a correlation coefficient of +0.92


b) Portfolio 3 & 4: with a correlation coefficient of +0.37
c) Portfolio 5 & 6: with a correlation coefficient of 0
d) Portfolio 7 & 8: with a correlation coefficient of -0.78

Question 21.
Which of the following assumptions support the use of technical analysis?
a) Future performance should be reflective of past performance
b) The values of market indices and stock prices are determined based on supply and
demand
c) Stock prices move in trends that would persist over long periods
d) All the above

Question 22.
Given the following information about securities A and B:
Historical Returns for Securities A B
Year 1 10% 18%
Year 2 6% 12%
Year 3 0% 2%

Which of the following are true about Securities A and B?

1) A is more risky because it has a higher standard deviation.


2) B is more risky because it has a higher standard deviation. .
3) A has a higher risk-adjusted return.
4) B has a higher risk-adjusted return.

a) 1 and 3
b) 1 and 4
c) 2 and 3
d) 2 and 4

Question 23.
What is the duration of a zero coupon bond with yield to maturity of 6% maturing in 6
years time?

a) 4.35
b) 5.34
c) 6.00
d) 6.35

Financial Planning Academy 142


Question 24.
In industry life cycle, revenue, margin and profit are expected to peak in the following
order:

a) Revenue, margin, profit


b) Margin, profit, revenue
c) Margin, revenue, profit
d) Profit, margin, revenue

Question 25.
The broadest measure of inflation is:

a) Consumer Price Index


b) Wholesale Price Index
c) Price deflator
d) Both a and b

Question 26.
If a bond is selling at a premium ______________________________

a) It is an attractive investment
b) Its realised compound yield will be less than the yield to maturity
c) Its coupon rate is below market rate
d) Its current yield is lower than the coupon rate

Question 27.
The trust deed lays down the terms and conditions under which the unit holders money is
to be invested. Specifically, it details:

a) The unit trust scheme


b) The types of authorized investments
c) All fees and charges
d) All of the above

Financial Planning Academy 143


Question 28.
Which of the following is NOT a characteristic of a balance fund?

a) It provides both growth and income objectives


b) It is less risky than growth funds
c) It is more risky than income funds
d) It must invest in both equity and bonds in equal amount

Question 29.
Disclosure statements to prospective clients include all of the following except:
a) Performance record of other clients
b) The method of remuneration, fees and commissions
c) Access to internal and external complaint handling mechanism
d) Disclosure of any conflict of interest

Financial Planning Academy 144


Solution :- ( Mock Test – IV )

Q1. The Correct answer C

Q2. The Correct answer C

Q3. The Correct answer D

Q4. The Correct answer B

Q5. The Correct answer C

Total number of shares before bonus issue: 800 shares


Total value of shares issued before bonus issue: 1,600 = 800 × 2.00
Bonus share entitlement: 200 shares = 800 shares ÷ 4
Total number of shares after bonus issue: 1,000 shares = 800 + 200
Price per share after bonus issue: 1.60 = 1,600 ÷ 1,000

Q6. The Correct answer D

Q7. The Correct answer C

Q8. The Correct answer C

Q9. The Correct answer B

Q10. The Correct answer D

Q11. The Correct answer A

Q12. The Correct answer D

Q13. The Correct answer C

Q14. The Correct answer A

Q15. The Correct answer B

Q16. The Correct answer D

Q17. The Correct answer D

Q18. The Correct answer A

Q19. The Correct answer C

Financial Planning Academy 145


Q20. The correct answer is D

Q21. The correct answer is D

Q22. The correct answer is D

Q23. The correct answer is C

Q24. The correct answer is B

Q25. The correct answer is C

Q26. The correct answer is D

Q27. The correct answer is D

Q28. The correct answer is D

Q29. The correct answer is A

Financial Planning Academy 146

You might also like